MedSurg/Evolve & Book Questions Test 3

¡Supera tus tareas y exámenes ahora con Quizwiz!

Chapter 56: Nursing Assessment Nervous System. Book/Evolve Questions

....

Chapter 57: Nursing Management: Acute Intracranial Problems. Book/Evolve Questions

....

Chapter 58: Nursing Management: Chronic Neurologic Problems

....

Chapter 58: Nursing Management: Stroke: Book & Evolve Questions:

.....

Chapter 60: Nursing Management: Alzheimer's Disease, & Delirium Book & Evolve Questions

.....

Chapter 65: Arthritis & Connective Tissue Diseases : Online Evolve & Book questions

.....

Chapter 61: Nursing Management: Peripheral Nerve and Spinal Cord Problems Book & Evolve Questions

......

Chapter 14 Altered Immune Responses & Transplantation: Evolve/Book Questions

..........

Chapter 11: Substance Abuse Evolve/Book Questionsf

...........

When admitting a patient to the emergency department who complains of chest pain, which assessment finding would alert the nurse to possible cocaine abuse? A. Insomnia B. Bradycardia C. Hypothermia D. Constricted pupils

Answer: A Insomnia Rationale: Insomnia and restlessness occur from the nervous system stimulation by the stimulant, cocaine. The nurse should suspect stimulant drug abuse in any patient seeking health care who has tachycardia, hyperactivity, fever, or behavioral abnormalities.

The nurse who has administered a dose of risperidone (Risperdal) to a patient with delirium should assess for what intended effect of the medication? A. Lying quietly in bed B. Alleviation of depression C. Reduction in blood pressure D. Disappearance of confusion

Answer: A Lying quietly in bed Rationale: Risperidone is an antipsychotic drug that reduces agitation and produces a restful state in patients with delirium. However, it should be used with caution. Antidepressant medications treat depression, and antihypertensive medications treat hypertension. However, there are no medications that will cause confusion to disappear in a patient with delirium.

A major goal of treatment for the patient with AD is to A. Maintain patient safety B. Maintain or increase body weight. C. Return to a higher level of self-care. D. Enhance functional ability over time.

Answer: A Maintain patient safety Rationale: The overall management goals are that the patient with AD will (1) maintain functional ability for as long as possible, (2) be maintained in a safe environment with a minimum of injuries, (3) have personal care needs met, and (4) have dignity maintained. The nurse should place emphasis on patient safety while planning and providing nursing care

Vasogenic cerebral edema increases intracranial pressure by A. Shifting fluid in the gray matter B. Altering the endothelial lining of cerebral capillaries C. Leaking molecules form the intracellular fluid to the capillaries D. Altering the osmotic gradient flow into the intravascular component.

Answer: B Altering the endothelial lining of cerebral capillaries Rationale: Vasogenic cerebral edema occurs mainly in the white matter. It is caused by changes in the endothelial lining of cerebral capillaries.

A patient is receiving chlordiazepoxide (Librium) to prevent the occurrence of delirium tremens. Which clinical manifestation would indicate an overdose of this medication? A. Bradycardia B. Hyperreflexia C. Warm, flushed skin D. Shallow respirations

Answer: D Shallow respirations Rationale: Chlordiazepoxide (Librium) is a sedative-hypnotic in the benzodiazepine family. Clinical manifestations of overdose include shallow respirations, hypotension; weak, rapid pulse; slow reflexes, impaired motor function, coma, and possible death.

How does an antigen stimulate an immune response? a. It is captured, processed, and presented to a lymphocyte by a macrophage. b. It circulates in the blood, where it comes in contact with circulating lymphocytes. c. It is a foreign protein that has antigenic determinants different from those of the body. d. It combines with larger molecules that are capable of stimulating production of antibodies.

Answer: A Both B and T lymphocytes must be sensitized by a processed antigen to activate the immune response. Processing involves the taking up of an antigen on the macrophage cell membrane, and presentation to the lymphocytes. Antigens do not need to be proteins and a few antigens may combine with larger molecules that are antigenic.

A 50-year-old man complains of recurring headaches. He describes these as a sharp, stabbing, and located around his left eye. He also reports that his left eye seems to swell and get teary when these headaches occur. Based on this history, you suspect that he has A. Cluster headaches B. Tension headaches C. Migraine headaches D. Medication overuse headaches

Answer: A Rationale: Cluster headaches involve repeated headaches that can occur for weeks to months, followed by periods of remission. The pain of cluster headache is sharp and stabbing; the intense pain lasts a few minutes to 3 hours. Cluster headaches can occur every other day and as often as eight times a day. The clusters occur with regularity, usually occurring at the same time each day and during the same seasons of the year. Typically, a cluster lasts 2 weeks to 3 months, and the patient then goes into remission for months to years. The pain usually is located around the eye and radiates to the temple, forehead, cheek, nose, or gums. Other manifestations may include swelling around the eye, lacrimation (tearing), facial flushing or pallor, nasal congestion, and constriction of the pupil. During the headache, the patient is often agitated and restless, unable to sit still or relax.

The nurse observes a student nurse assigned to initiate oral feedings for a 68-year-old woman with an ischemic stroke. The nurse should intervene if she observes the student nurse A. giving the patient 8 ounces of ice water to swallow. B. telling the patient to perform a chin tuck before swallowing. C. assisting the patient to sit in a chair before feeding the patient. D. assessing cranial nerves IX and X before the patient attempts to eat.

Answer: A Rationale: The majority of patients after a stroke have dysphagia. The gag reflex and swallowing ability (cranial nerves IX and X) should be assessed before the first oral feeding. To assess swallowing ability, the nurse should elevate the head of the bed to an upright position (unless contraindicated) and give the patient a very small amount (not 8 ounces) of crushed ice or ice water to swallow. The patient should remain in a high Fowler's position, preferably in a chair with the head flexed forward, for the feeding and for 30 minutes following.

The nurse performs discharge teaching for a 34-year-old male patient with a T2 spinal cord injury resulting from a construction accident. Which statement, if made by the patient to the nurse, indicates that teaching about recognition and management of autonomic dysreflexia is successful? A. "I will perform self-catheterization at least six times per day." B. "A reflex erection may cause an unsafe drop in blood pressure." "If I develop a severe headache, I will lie down for 15 to 20 minutes." "I can avoid this problem by taking medications to prevent leg spasms."

Answer: A "I will perform self-catheterization at least six times per day." Rationale: Autonomic dysreflexia is usually caused by a distended bladder. Performing self-catheterization five or six times a day prevents bladder distention. Signs and symptoms of autonomic dysreflexia include a severe headache. Patients should raise the head of the bed to 45 to 90 degrees. This action helps to relieve hypertension (systolic pressure up to 300 mm Hg) that occurs with autonomic dysreflexia.

Which statement by the wife of a patient with Alzheimer's disease (AD) demonstrates an accurate understanding of her husband's medication regimen? A. "I'm really hoping his medications will slow down his mental losses." B. "We're both holding out hope that this medication will cure his disease." C. "I know that this won't cure him, but we learned that it might prevent a bodily decline while he declines mentally." D. "I learned that if we are vigilant about his medication schedule, he may not experience the physical effects of his disease."

Answer: A "I'm really hoping his medications will slow down his mental losses." Rationale: There is presently no cure for Alzheimer's disease, and drug therapy aims at improving or controlling decline in cognition. Medications do not directly address the physical manifestations of AD.

A 24-year-old female patient with systemic lupus erythematosus (SLE) tells the nurse she wants to have a baby and is considering getting pregnant. Which response by the nurse is most appropriate? A. "Infertility can result from the medications used to control your disease." B. "Pregnancy will result in a temporary remission of your signs and symptoms." C. "Autoantibodies transferred to the baby during pregnancy will cause heart defects." D. "The baby is at high risk for neonatal lupus erythematosus being diagnosed at birth."

Answer: A "Infertility can result from the medications used to control your disease." Rationale: Infertility may be caused by renal involvement and the previous use of high-dose corticosteroid and chemotherapy drugs. Neonatal lupus erythematosus rarely occurs in infants born to women with SLE. Exacerbation is common following pregnancy during the postpartum period. Spontaneous abortion, stillbirth, and intrauterine growth retardation are common problems with pregnancy related to deposits of immune complexes in the placenta and because of inflammatory responses in the placental blood vessels. There is not an increased risk for heart defects.

3. The reason newborns are protected for the first 6 months of life from bacterial infections is because of the maternal transmission of A. IgG B. IgA C. IgM D. IgE

Answer: A IgG Rationale: Immunoglobulin G (IgG) crosses the placental membrane and provides the newborn with passive acquired immunity for at least 3 months. Infants also may obtain some passive immunity from immunoglobulin A (IgA) in breast milk and colostrum.

In which patient would it be the most important for the nurse to assess the glossopharyngeal and vagus nerves? A. A 50-year-old woman with lethargy from a drug overdose B. A 40-year-old man with a complete lumbar spinal cord injury C. A 60-year-old man with severe pain from trigeminal neuralgia D. A 30-year-old woman with a high fever and bacterial meningitis

Answer: A A 50-year-old woman with lethargy from a drug overdose Rationale: The glossopharyngeal and vagus nerves innervate the pharynx and are tested by the gag reflex. It is important to assess the gag reflex in patients who have a decreased level of consciousness, a brainstem lesion, or a disease involving the throat musculature. If the reflex is weak or absent, the patient is in danger of aspirating food or secretions.

A patient with a suspected traumatic brain injury has bloody nasal drainage. What observation should cause the nurse to suspect that this patient has a cerebrospinal fluid (CSF) leak? A. A halo sign on the nasal drip pad B. Decreased blood pressure and urinary output C. A positive reading for glucose on a Test-tape strip D. Clear nasal drainage along with the bloody discharge

Answer: A A halo sign on the nasal drip pad Rationale: When drainage containing both CSF and blood is allowed to drip onto a white pad, within a few minutes the blood will coalesce into the center, and a yellowish ring of CSF will encircle the blood, giving a halo effect. The presence of glucose would be unreliable for determining the presence of CSF because blood also contains glucose. Decreased blood pressure and urinary output would not be indicative of a CSF leak.

For a 65-year-old woman who has lived with a T1 spinal cord injury for 20 years, which health teaching instructions should the nurse emphasize? A. A mammogram is needed every year B. Bladder function tends to improve with age. C. Heart disease is not common in persons wit spinal cord injury. D. As a person ages, the need to change body position is less important

Answer: A A mammogram is needed every year Rationale: Health promotion and screening are important for an older patient with a spinal cord injury. Older adult women with spinal cord injuries should perform monthly breast examinations and undergo yearly mammography.

A 63-year-old woman is admitted via the emergency department with a fractured pelvis and femur after a motor vehicle crash. The patient reports severe pain which she rates as "10" on a scale of 0-10. Health records indicate that the patient has disclosed an opioid addiction. How will the nurse plan to manage this patient's pain? A. Administer opioids at a higher dose than for drug-naïve patients. B. Administer opioids sparingly if the patient agrees to substance abuse counseling. C. Non-narcotic pain medication should be administered to reduce drug-seeking behaviors. D. A mixed opioid agonist-antagonist such as butorphanol (Stadol) should be administered.

Answer: A Administer opioids at a higher dose than for drug-naïve patients. Rationale: Severe pain in a patient with an opioid addiction should be treated with opioids, and at much higher doses than those used with drug-naïve patients. Non-opioid medication is unlikely to be sufficient to manage the pain. A mixed opioid agonist-antagonist should be avoided because this may precipitate withdrawal symptoms.

The nurse reminds the staff that standard precautions should be used when providing care for which type of patient? A. All patients regardless of diagnosis B. Pediatric and gerontologic patients C. Patients who are immunocompromised D. Patients with a history of infectious diseases

Answer: A All patients regardless of diagnosis Rationale: Standard precautions are designed for all care of all patients in hospitals and health care facilities.

A 19-year-old woman is hospitalized for a frontal skull fracture from a blunt force head injury. Clear fluid is draining from the patient's nose. What action by the nurse is most appropriate? A. Apply a loose gauze pad under the patient's nose. B. Place the patient in a modified Trendelenburg position. C. Ask the patient to gently blow the nose to clear the drainage. D. Gently insert a catheter in the nares and suction the drainage.

Answer: A Apply a loose gauze pad under the patient's nose. Rationale: Cerebrospinal fluid (CSF) rhinorrhea (clear or bloody drainage from the nose) may occur with a frontal skull fracture. If CSF rhinorrhea occurs, the nurse should inform the physician immediately. A loose collection pad may be placed under the nose. The head of the bed may be raised to decrease the CSF pressure so that a tear can seal. The nurse should not place a dressing or tube in the nasal cavity, and the patient should not sneeze or blow the nose.

The nurse is completing a health assessment for an obese 62-year-old man who wants to begin a diet and exercise program. Which assessment should the nurse perform to determine the cognitive function of the patient during the physical examination? A. Ask the patient a question such as, "Who were the last three presidents?" B. Determine the level of consciousness, body posture, and facial expressions. C. Observe for signs of agitation, anger, or depression during the health check. D. Request that the patient mimic rapid, alternating movements with both hands.

Answer: A Ask the patient a question such as, "Who were the last three presidents?" Rationale: Cognition is one component of the mental status examination to determine cerebral functioning. Cognition is assessed by determining orientation, memory, general knowledge, insight, judgment, problem solving, and calculation. A question often used to determine cognition for adults living in the United States is, "Who were the last three presidents?" General appearance and behavior is another component and includes level of consciousness, body posture, and facial expressions. Mood and affect are assessed by observing for agitation, anger, or depression. Cerebellar function is determined by assessing balance and coordination and may include testing rapid alternating movements of the upper and lower extremities.

The nurse cares for a 34-year-old woman after a lumbar puncture. Which action by the nurse is most appropriate? A. Assess for drainage or bleeding from the puncture site. B. Monitor for bladder dysfunction and bowel incontinence. C. Maintain bed rest until lower extremities move normally. D. Check for loss of muscle strength in the upper extremities.

Answer: A Assess for drainage or bleeding from the puncture site. Rationale: After a lumbar puncture the nurse should monitor the puncture site for drainage or bleeding. Other assessments include headache intensity, meningeal irritation (nuchal rigidity), signs and symptoms of local trauma (e.g., hematoma, pain), neurologic signs, and vital signs. A lumbar puncture does not affect bowel or bladder function or upper extremity muscle strength. Bed rest until lower extremity movement returns is indicated for the patient after spinal anesthesia.

A patient who has a neurologic disease that affects the pyramidal tract is likely to manifest what sign? A. Impaired muscle movement B. Decreased deep tendon reflexes C. Decreased level of consciousness D. Impaired sensation of touch, pain, and temperature

Answer: A Impaired muscle movement Rationale:

When assessing a patient with a traumatic brain injury, the nurse notes uncoordinated movement of the extremities. How should the nurse document this assessment? A. Ataxia B. Apraxia C. Anisocoria D. Anosognosia

Answer: A Ataxia Rationale: Ataxia is a lack of coordination of movement, possibly caused by lesions of sensory or motor pathways, cerebellum disorders, or certain medications. Apraxia is the inability to perform learned movements despite having the desire and physical ability to perform them related to a cerebral cortex lesion. Anisocoria is inequality of pupil size from an optic nerve injury. Anosognosia is the inability to recognize a bodily defect or disease related to lesions in the right parietal cortex. b

On initial assessment of an older patient, the nurse knows to look for certain types of diseases because which immunologic response increases with age? A. Autoimmune response B. Cell-mediated immunity C. Hypersensitivity response D. Humoral immune response

Answer: A Autoimmune response Rationale: With aging, autoantibodies increase, which lead to autoimmune diseases (e.g., systemic lupus erythematosus, acute glomerulonephritis, rheumatoid arthritis, hypothyroidism). Cell-mediated immunity decreases with decreased thymic output of T cells and decreased activation of both T and B cells. There is a decreased or absent delayed hypersensitivity reaction. Immunoglobulin levels decrease and lead to a suppressed humoral immune response in older adults.

Which clinical manifestation would the nurse interpret as a manifestation of neurogenic shock in a patient with acute spinal cord injury? A. Bradycardia B. Hypertension C. Neurogenic spasticity D. Bounding pedal pulses

Answer: A Bradycardia Rationale: Neurogenic shock is due to the loss of vasomotor tone caused by injury and is characterized by bradycardia and hypotension. Loss of sympathetic innervations causes peripheral vasodilation, venous pooling, and a decreased cardiac output. Thus hypertension, neurogenic spasticity, and bounding pedal pulses are not seen in neurogenic shock.

The HIV-infected patient is taught health promotion activities including good nutrition; avoiding alcohol, tobacco, drug use, and exposure to infectious agents; keeping up to date with vaccines; getting adequate rest; and stress management. What is the rationale behind these interventions that the nurse knows? A. Delaying disease progression B. Preventing disease transmission C. Helping to cure the HIV infection D. Enabling an increase in self-care activities

Answer: A Delaying disease progression Rationale: These health promotion activities along with mental health counseling, support groups, and a therapeutic relationship with health care providers will promote a healthy immune system, which may delay disease progression. These measures will not cure HIV infection, prevent disease transmission, or increase self-care activities.

9. The most common cause of secondary immunodeficiencies is A. Drugs B. Stress C. Malnutrition D. Human immunodeficiency virus.

Answer: A Drugs Rationale: Drug-induced immunosuppression is the most common cause of secondary immunodeficiency disorders.

5. The nurse is alerted to possible anaphylactic shock immediately after a patient has received intramuscular penicillin by the development of A. Edema and itching at the injection site. B. Sneezing and itching of the nose and eyes. C. A wheal-and-flare reaction at the injection site. D. Chest tightness and production of thick sputum.

Answer: A Edema and itching at the injection site. Rationale: Rationale: Initial symptoms include edema and itching at the site of the exposure to the allergen

A patient is suspected of having a brain tumor. The signs and symptoms include memory deficits, visual disturbances, weakness of right upper and lower extremities, and personality changes. The nurse recognizes that the tumor is most likely located in the A. Frontal lobe B. Parietal lobe C. Occipital lobe D. Temporal lobe

Answer: A Frontal lobe Rationale: A unilateral frontal lobe tumor may result in the following signs and symptoms: unilateral hemiplegia, seizures, memory deficit, personality and judgment changes, and visual disturbances. A bilateral frontal lobe tumor may cause symptoms associated with a unilateral frontal lobe tumor and an ataxic gait.

Although he has been told that ginkgo biloba will probably have no effect, a 58-year-old man with early stage Alzheimer's disease insists on taking the herb because he believes it will slow the disease progression. Which statement, if made by the patient to the nurse, indicates understanding about the side effects of ginkgo? A. "Ginkgo may increase the risk of bruising." B. "Ginkgo may cause leg pain while walking." C. "It is not safe to suddenly stop taking ginkgo." D. "Ringing in the ears is a side effect of ginkgo."

Answer: A Ginkgo may increase the risk of bruising." Rationale: Ginkgo biloba may increase the risk for bruising and bleeding. There are no indications that sudden withdrawal of ginkgo biloba is unsafe. Ginkgo biloba is possibly effective for treating intermittent claudication (leg pain while walking). There is insufficient evidence to indicate that ginkgo biloba is effective in treatment of tinnitus (ringing in the ears).

A 34-year-old female patient who has systemic lupus erythematosus is receiving plasmapheresis to treat an acute attack. What symptoms will the nurse monitor to determine if the patient develops complications related to the procedure? A. Hypotension, paresthesias, and dizziness Correct B. Polyuria, decreased reflexes, and lethargy Incorrect C. Intense thirst, flushed skin, and weight gain D. Abdominal cramping, diarrhea, and leg weakness

Answer: A Hypotension, paresthesias, and dizziness Rationale: Common complications associated with plasmapheresis are hypotension and citrate toxicity. Citrate is used as an anticoagulant and may cause hypocalcemia, which may manifest as headache, paresthesias, and dizziness. Polyuria, decreased reflexes, and lethargy are symptoms of hypercalcemia. Abdominal cramping, diarrhea, and leg weakness indicate hyperkalemia. Intense thirst, flushed skin, and weight gain indicate hypernatremia with normal or increased extracellular fluid volume.

The patient with type 1 diabetes mellitus with hypoglycemia is having a seizure. Which medication should the nurse anticipate administering to stop the seizure? A. IV dextrose solution B. IV diazepam (Valium) C. IV phenytoin (Dilantin) D. Oral carbamazepine (Tegretol)

Answer: A IV dextrose solution Rationale: This patient's seizure is caused by low blood glucose, so IV dextrose solution should be given first to stop the seizure. IV diazepam, IV phenytoin, and oral carbamazepine would be used to treat seizures from other causes such as head trauma, drugs, and infections.

Computed tomography (CT) of a 68-year-old male patient's head reveals that he has experienced a hemorrhagic stroke. What is the priority nursing intervention in the emergency department? A. Maintenance of the patient's airway B. Positioning to promote cerebral perfusion C. Control of fluid and electrolyte imbalances D. Administration of tissue plasminogen activator (tPA)

Answer: A Maintenance of the patient's airway Rationale: Maintenance of a patent airway is the priority in the acute care of a patient with a hemorrhagic stroke. It supersedes the importance of fluid and electrolyte imbalance and positioning. tPA is contraindicated in hemorrhagic stroke.

Data regarding mobility, strength, coordination, and activity tolerance are important for the nurse to obtain because A. Many neurologic diseases affect one or more of these areas. B. Patients are less able to identify other neurologic impairments. C. These are the first functions to be affected by neurologic diseases. D. Aspects of movement are the most important function of the nervous system.

Answer: A Many neurologic diseases affect one or more of these areas. Rationale: Many neurologic disorders affect the patient's mobility, strength, and coordination. These problems can alter the patient's usual activity and exercise patterns.

What nursing intervention should be implemented in the care of a patient who is experiencing increased ICP? A. Monitor fluid and electrolyte status carefully. B. Position the patient in a high Fowler's position. C. Administer vasoconstrictors to maintain cerebral perfusion. D. Maintain physical restraints to prevent episodes of agitation.

Answer: A Monitor fluid and electrolyte status carefully Rationale: Fluid and electrolyte disturbances can have an adverse effect on ICP and must be monitored vigilantly. The head of the patient's bed should be kept at 30 degrees in most circumstances, and physical restraints are not applied unless absolutely necessary. Vasoconstrictors are not typically administered in the treatment of ICP.

9. A patient's eyes jerk while the patient looks to the left. you will record this finding as A. Nystagmus B. CN VI palsy C. Oculocephalia D. Ophthalmic dyskinesia

Answer: A Nystagmus Rationale: Nystagmus is defined as fine, rapid jerking movements of the eyes.

The nurse is caring for four newly diagnosed patients with various connective tissue disorders. The nurse should be most aware of safety issues and interstitial lung involvement in the patient with which diagnosis? A. Polymyositis B. Reactive arthritis C. Sjögren's syndrome D. Systemic lupus erythematosus (SLE)

Answer: A Polymyositis Rationale: Polymyositis is an inflammatory disease affecting striated muscle and resulting in muscle weakness that increases the patient's risk of falls and injury. Weakened pharyngeal muscles increase the risk for aspiration with interstitial lung disease in up to 65% of patients. The treatment of polymyositis starts with high-dose corticosteroids that cause immunosuppression. If this does not work, other immunosuppressive drugs may be used. Reactive arthritis (Reiter's syndrome) occurs with urethritis, conjunctivitis, and mucocutaneous lesions with the asymmetric arthritis involving large joints of the lower extremities and toes. This patient is not at increased risk for safety problems. Sjögren's syndrome decreases moisture produced by exocrine glands, especially in the mouth and eyes and is without increased risk of injury or interstitial lung involvement. Systemic lupus erythematosus (SLE) is a multisystem inflammatory autoimmune disorder treated with NSAIDs, antimalarial agents. Safety would not be an important issue early in the disease.

A 48-year-old man was just diagnosed with Huntington's disease. His 20-year-old son is upset about his father's diagnosis. How can the nurse best help this young man? A. Provide emotional and psychologic support. B. Encourage him to get diagnostic genetic testing done. C. Tell him the cognitive deterioration will be treated with counseling. D. Tell him the chorea and psychiatric disorders can be treated with haloperidol (Haldol).

Answer: A Provide emotional and psychologic support. Rationale: The patient's son will first need emotional and psychologic support. He should be taught about diagnostic genetic testing for himself but should decide for himself with a genetic counselor if and when he wants this done. The treatment plan for his father will be determined depending on his father's needs.

A patient with heart failure and type 1 diabetes mellitus is scheduled for a positron emission tomography (PET) of the brain. Which medication prescribed by the health care provider should the nurse expect to administer before the diagnostic study? A. Regular insulin 6 units (SQ) B. Furosemide (Lasix) 20 mg (IV) C. Alprazolam (Xanax) 0.5 mg (PO) D. Ciprofloxacin (Cipro) 500 mg (PO)

Answer: A Regular insulin 6 units (SQ) Rationale: Patients with type 1 diabetes mellitus must have insulin administered the day of the PET study if glucose metabolism is the focus of the PET. Diuretics should not be administered before the PET scan unless a urinary catheter is inserted. The patient must remain still during the procedure (1 to 2 hours). Sedatives and tranquilizers (e.g., alprazolam) should not be administered before a PET study of the brain because the patient may need to perform mental activities and these medications may affect glucose metabolism. Prophylactic antibiotics are not necessary. Patients are NPO before a PET study of the brain and should not receive oral medications (alprazolam and ciprofloxacin).

The nurse should teach the patient with ankylosing spondylitis the importance of A. Regularly exercising and maintaining proper posture. B. Avoiding extremes in environmental temperatures. C. Maintaining usual physical activity during flare-ups. D. Applying hot and cool compresses for relief of local symptoms.

Answer: A Regularly exercising and maintaining proper posture. Rationale: Patients with ankylosing spondylitis (AS) should exercise after pain and stiffness are managed. Postural control is important for minimizing spinal deformity. The exercise regimen should include back, neck, and chest stretches. The nurse should educate the patient with AS about regular exercise and attention to posture, local moist-heat applications, and knowledgeable use of drugs. The nurse should discourage excessive physical exertion during periods of active flare-up of the disease. Proper positioning at rest is essential. The mattress should be firm, and the patient should sleep on their back with a flat pillow, avoiding positions that encourage flexion deformity. Postural training emphasizes avoiding spinal flexion (e.g., leaning over a desk), heavy lifting, and prolonged walking, standing, or sitting.

A 78-year-old woman is in the intensive care unit after emergency abdominal surgery. The nurse notes that the patient is disoriented and confused, has incoherent speech, and is restless and agitated. Which action by the nurse is most appropriate? A. Reorient the patient. B. Notify the physician. C. Document the findings. D. Administer lorazepam (Ativan).

Answer: A Reorient the patient. Rationale: The patient is exhibiting clinical manifestations of delirium. Care of the patient with delirium is focused on eliminating precipitating factors and protecting the patient from harm. Give priority to creating a calm and safe environment. The nurse should stay at the bedside and provide reassurance and reorienting information as to place, time, and procedures. The nurse should reduce environmental stimuli, including noise and light levels. Avoid the use of chemical and physical restraints if possible.

The patient with diabetes mellitus has had a right-sided stroke. Which nursing intervention should the nurse plan to provide for this patient related to expected manifestations of this stroke? A. Safety measures B. Patience with communication C. Mobility assistance on the right side D. Place food in the left side of patient's mouth.

Answer: A Safety measures A patient with a right-sided stroke has spatial-perceptual deficits, tends to minimize problems, has a short attention span, is impulsive, and may have impaired judgment. Safety is the biggest concern for this patient. Hemiplegia occurs on the left side of this patient's body. The patient with a left-sided stroke has hemiplegia on the right, is more likely to have communication problems, and needs mobility assistance on the right side with food placed on the left side if the patient needs to be fed after a swallow evaluation has taken place.

Magnetic resonance imaging (MRI) has revealed the presence of a brain tumor in a patient. The nurse should recognize that the patient will most likely need which treatment modality? A. Surgery B. Chemotherapy C. Radiation therapy D. Biologic drug therapy

Answer: A Surgery Rationale: Surgical removal is the preferred treatment for brain tumors. Chemotherapy and biologic drug therapy are limited by the blood-brain barrier, tumor cell heterogeneity, and tumor cell drug resistance. Radiation therapy may be used as a follow-up measure after surgery.

A 62-year-old patient has acquired immunodeficiency syndrome (AIDS), and the viral load is reported as undetectable. What patient teaching should be provided by the nurse related to this laboratory study result? A. The patient has the virus present and can transmit the infection to others. B. The patient is not able to transmit the virus to others through sexual contact. C. The patient will be prescribed lower doses of antiretroviral medications for 2 months. D. The syndrome has been cured, and the patient will be able to discontinue all medications.

Answer: A The patient has the virus present and can transmit the infection to others. Rationale: In human immunodeficiency virus (HIV) infections, viral loads are reported as real numbers of copies/μL or as undetectable. "Undetectable" indicates that the viral load is lower than the test is able to report. "Undetectable" does not mean that the virus has been eliminated from the body or that the individual can no longer transmit HIV to others.

Transmission of HIV from an infected individual to another most commonly occurs as a result of A. Unprotected anal or vaginal sexual intercourse. B. Low levels of virus in the blood and high levels of CDR T cells. C. Sharing from mother to infant during labor and delivery and breastfeeding. D. Sharing of drug-using equipment, including needles, syringes, pipes, and straws.

Answer: A Unprotected anal or vaginal sexual intercourse Rationale: Unprotected sexual contact (semen, vaginal secretions, or blood) with a partner infected with human immunodeficiency virus (HIV) is the most common mode of HIV transmission.

An 82-year-old woman is brought to her physician by her daughter with complaints of some confusion. What testing should the nurse suggest for this patient? A. Urinalysis B. Sputum culture C. Red blood cell count D. White blood cell count

Answer: A Urinalysis Rationale: The developments of urinary tract infections commonly contribute to atypical manifestations such as cognitive and behavior changes in older adults. Sputum culture, red blood cell count, and white blood cell count may be done, but the first step would be to assess for a possible urinary tract infection.

Which intervention should the nurse perform in the acute care of a patient with autonomic dysreflexia? A. Urinary catheterization B. Administration of benzodiazepines C. Suctioning of the patient's upper airway D. Placement of the patient in the Trendelenburg position

Answer: A Urinary catheterization Rationale: Because the most common cause of autonomic dysreflexia is bladder irritation, immediate catheterization to relieve bladder distention may be necessary. Benzodiazepines are contraindicated, and suctioning is likely unnecessary. The patient should be positioned upright.

Which strategy can the nurse teach the patient to eliminate the risk of HIV transmission? A. Using sterile equipment to inject drugs B. Cleaning equipment used to inject drugs C. Taking zidovudine (AZT, ZDV, Retrovir) during pregnancy D. Using latex or polyurethane barriers to cover genitalia during sexual contact

Answer: A Using sterile equipment to inject drugs Rationale: Access to sterile equipment is an important risk-elimination tactic. Some communities have needle and syringe exchange programs (NSEPs) that provide sterile equipment to users in exchange for used equipment. Cleaning equipment before use is a risk-reducing activity. It decreases the risk when equipment is shared, but it takes time, and a person in drug withdrawal may have difficulty cleaning equipment.

Which antibiotic-resistant organisms cannot be killed by normal soap? A. Vancomycin-resistant enterococci B. Methicillin-resistant Staphylococcus aureus C. Penicillin-resistant Streptococcus pneumoniae D. B-Lactamase-producing Klebsiella pneumoniae

Answer: A Vancomycin-resistant enterococci Rationale: Vancomycin-resistant enterococci (VRE) can remain viable on environmental surfaces for weeks. An antiseptic soap such as chlorhexidine is needed to kill these bacteria.

Which measure should the nurse prioritize when providing care for a patient with a diagnosis of multiple sclerosis (MS)? A. Vigilant infection control and adherence to standard precautions B. Careful monitoring of neurologic assessment and frequent reorientation C. Maintenance of a calorie count and hourly assessment of intake and output D. Assessment of blood pressure and monitoring for signs of orthostatic hypotension

Answer: A Vigilant infection control and adherence to standard precautions Rationale: Infection control is a priority in the care of patients with MS, since infection is the most common cause of an exacerbation of the disease. Decreases in cognitive function are less likely, and MS does not typically result in malnutrition, hypotension, or fluid volume excess or deficit.

When teaching a patient infected with HIV regarding transmission of the virus to others, which statement made by the patient would indicate a need for further teaching? A. "I will need to isolate any tissues I use so as not to infect my family." B. "I will notify all of my sexual partners so they can get tested for HIV." C. "Unprotected sexual contact is the most common mode of transmission." D. "I do not need to worry about spreading this virus to others by sweating at the gym."

Answer: A "I will need to isolate any tissues I use so as not to infect my family." Rationale: HIV is not spread casually. The virus cannot be transmitted through hugging, dry kissing, shaking hands, sharing eating utensils, using toilet seats, or attending school with an HIV-infected person. It is not transmitted through tears, saliva, urine, emesis, sputum, feces, or sweat.

For which patient should the nurse prioritize an assessment for depression? A. A patient in the early stages of Alzheimer's disease B. A patient who is in the final stages of Alzheimer's disease C. A patient experiencing delirium secondary to dehydration D. A patient who has become delirious following an atypical drug response

Answer: A A patient in the early stages of Alzheimer's disease Answer: Patients in the early stages of Alzheimer's disease are particularly susceptible to depression, since the patient is acutely aware of his or her cognitive changes and the expected disease trajectory. Delirium is typically a shorter-term health problem that does not typically pose a heightened risk of depression.

A diagnosis of AIDS is made when an HIV-infected patient has A. CD4 T cell count below 200/uL B. A high level of HIV in the blood and saliva. C. Lipodystrophy with metabolic abnormalities. D. Oral hair leukoplakia, an infection caused by Epstein-Barr virus.

Answer: A CD4 T cell count below 200/uL Rationale: AIDS is diagnosed when an individual with HIV infection meets one of several criteria; one criterion is a CD4+ T cell count below 200 cells/L. Other criteria are listed in Table 15-9.

The nurse teaches a 64-year-old man with gouty arthritis about food that may be consumed on a low-purine diet. Which food item, if selected by the patient, would indicate an understanding of the instructions? A. Eggs B. Liver C. Salmon D. Chicken

Answer: A Eggs Rationale: Gout is caused by an increase in uric acid production, underexcretion of uric acid by the kidneys, or increased intake of foods containing purines, which are metabolized to uric acid by the body. Liver is high in purine, and chicken and salmon are moderately high in purine.

Which manifestations in a patient with a T4 spinal cord injury should alert the nurse to the possibility of autonomic dysreflexia? A. Headache and rising blood pressure B. Irregular respirations and shortness of breath C. Decreased level of consciousness or hallucinations D. Abdominal distention and absence of bowel sounds

Answer: A Headache and rising blood pressure Rationale: Manifestations of autonomic dysreflexia are hypertension (up to 300 mm Hg systolic), a throbbing headache, bradycardia, and diaphoresis. Respiratory manifestations, decreased level of consciousness, and gastrointestinal manifestations are not characteristic manifestations.

Which modifiable risk factors for stroke would be most important for the nurse to include when planning a community education program? A. Hypertension B. Hyperlipidemia C. Alcohol consumption D. Oral contraceptive use

Answer: A Hypertension Rationale: Hypertension is the single most important modifiable risk factor, but it is still often undetected and inadequately treated. The public is often more aware of hyperlipidemia and oral contraceptive use as risk factors for stroke. Alcohol is also a modifiable risk factor.

During admission of a patient with a severe head injury to the emergency department, the nurse places the highest priority on assessment for A. Patency of airway B. Presence of neck injury C. Neurologic status with the Glasgow Coma Scale D. Cerebrospinal fluid leakage form the ears or nose.

Answer: A Patency of airway Rationale: The nurse's initial priority in the emergency management of a patient with a severe head injury is to ensure that the patient has a patent airway.

The patient is describing his use of an illicit drug as needing a larger dose to get the same effect. How should the nurse document this patient's usage? A. Tolerance B. Addiction C. Substance abuse D. Substance dependence

Answer: A Tolerance Rationale: Tolerance is the need for a larger dose of drug to obtain the original effect. Addiction is compulsive, uncontrollable dependence on a substance such that cessation causes severe emotional, mental, or physiologic reactions. Substance abuse is overindulgence in a substance that has a negative impact on the psychologic, physiologic, and/or social functioning of the individual. Substance dependence is such reliance on a substance that the absence of the substance will cause impairment in function.

Which characteristic describes immunoglobulin E (select all that apply)? a. Assists in parasitic infections b. Responsible for allergic rxns c. Present on the lymphocyte surface d. Assists in B-lymphocyte differentiation e. Predominant in secondary immune response f. Protects body surfaces and mucous membranes

Answer: A, B Immunoglobulin E (IgE) assists in parasitic infections and causes allergic rxns. IgA protects body surfaces and mucous membranes. IgD assists in B-lymphocyte differentiation and is present on the lymphocyte surface. IgG is predominant in the secondary immune response.

Teach the patient with fibromyalgia the importance of limiting intake of which foods (select all that apply)? A. Sugar B. Alcohol C. Caffeine D. Red meat E. Root Vegetables

Answer: A, B, C Sugar, Alcohol, Caffeine Rationale: Dietitians often urge patients with fibromyalgia to limit their consumption of sugar, caffeine, and alcohol because these substances have been shown to be muscle irritants.

Which statements accurately describe HIV infection (select all that apply)? A. Untreated HIV infection has a predictable pattern of progression. B. Late chronic HIV infection is called acquired immunodeficiency syndrome (AIDS). C. Untreatable HIV infection can remain in the early chronic stage for a decade or more. D. Untreated HIV infection usually remains in the early chronic stage for 1 year or less. E. Opportunistic diseases occur more often when the CD4 T cell count is high and the viral load is low.

Answer: A, B, C Untreated HIV infection has a predictable pattern of progression, Late chronic HIV infection is called acquired immunodeficiency syndrome (AIDS), & Untreatable HIV infection can remain in the early chronic stage for a decade or more. Rationale: The typical course of untreated HIV infection follows a predictable pattern. However, treatment can significantly alter this pattern, and disease progression is highly individualized. Late chronic infection is another term for acquired immunodeficiency syndrome (AIDS). The median interval between untreated HIV infection and a diagnosis of AIDS is about 11 years.

What should the nurse teach the patients in the assisted living facility to decrease their risk for antibiotic-resistant infection (select all that apply)? A. Wash hands frequently. B. Take antibiotics as prescribed. C. Take the antibiotic until it is gone. D. Take antibiotics to prevent illnesses like colds. E. Save leftover antibiotics to take if needed later.

Answer: A, B, C Wash hands frequently, Take antibiotics as prescribed, & Take the antibiotic until it is gone. Rationale: To decrease the risk for antibiotic-resistant infections, people should wash their hands frequently, follow the directions when taking the antibiotics, finish the antibiotic, do not request antibiotics for colds or flu, do not save leftover antibiotics, or take antibiotics to prevent an illness without them being prescribed by a health care provider.

Emerging infections can affect health care by (select all that apply) A. Revealing antibiotic resistance. B. Generating scientific discoveries. C. Creating a strain on limited resources. D. Challenging established medical traditions. E. Limiting travel options for nursing personnel

Answer: A, B, C, D Revealing antibiotic resistance, Generating scientific discoveries, Creating a strain on limited resources, & Challenging established medical traditions. Rationale: An emerging infection is an infectious disease whose incidence has increased in the past 20 years or threatens to increase in the immediate future. Examples include drug-resistant organisms that emerged when a previously treatable organism developed resistance to antibiotics; infections for which criteria for starting antiretroviral drug therapy (ART) have changed with new scientific discoveries; and infections for which obtaining adequate treatment is difficult because of limited resources or access.

The patient is having some increased memory and language problems. What diagnostic tests will be done before this patient is diagnosed with Alzheimer's disease (select all that apply)? A. Urinalysis B. MRI of the head C. Liver function tests D. Neuropsychologic testing E. Blood urea nitrogen and serum creatinine

Answer: A, B, C, D, E Rationale: Because there is no definitive diagnostic test for Alzheimer's disease, and many conditions can cause manifestations of dementia, testing must be done to eliminate any other causes of cognitive impairment. These include urinalysis to eliminate a urinary tract infection, an MRI to eliminate brain tumors, liver function tests to eliminate encephalopathy, BUN and serum creatinine to rule out renal dysfunction, and neuropsychologic testing to assess cognitive function.

Social effects of a chronic neurologic disease include (select all that apply) A. Divorce B. Job loss C. Depression D. Role changes. E. loss of self-esteem

Answer: A, B, C, D, E Rationale: problems related to chronic neurologic disease may include changes in roles and relationships (e.g., divorce, job loss, role changes); other psychologic problems (e.g., depression, loss of self-esteem) also may have social effects.

The nurse is caring for a patient admitted for evaluation and surgical removal of a brain tumor. The nurse will plan interventions for this patient based on knowledge that brain tumors can lead to which complications (select all that apply)? A. Vision loss B. Cerebral edema C. Pituitary dysfunction D. Parathyroid dysfunction E. Focal neurologic deficits

Answer: A, B, C, E Vision loss, cerebral edema, pitutiary dysfunciton, & focal neurologic deficits Rationale: Brain tumors can manifest themselves in a wide variety of symptoms depending on location, including vision loss and focal neurologic deficits. Tumors that put pressure on the pituitary can lead to dysfunction of the gland. As the tumor grows, clinical manifestations of increased intracranial pressure (ICP) and cerebral edema appear. The parathyroid gland is not regulated by the cerebral cortex or the pituitary gland.

The patient with fibromyalgia is suffering with pain at 12 of the 18 identification sites, including the neck and upper back and the knees. The patient also reports nonrefreshing sleep, depression, and being anxious when dealing with multiple tasks. The nurse should teach this patient about what treatments (select all that apply)? A. Low-impact aerobic exercise B. Relaxation strategy (biofeedback) C. Antiseizure drug pregabalin (Lyrica) D. Morphine sulfate extended-release tablets E. Serotonin reuptake inhibitor (e.g., sertraline [Zoloft])

Answer: A, B, C, E Low-impact aerobic exercise, Relaxation strategy (biofeedback), Antiseizure drug pregabalin (Lyrica), Serotonin reuptake inhibitor (e.g., sertraline [Zoloft]) Rationale: Because the treatment of fibromyalgia is symptomatic, this patient will be prescribed something for pain, such as pregabalin, and a serotonin reuptake inhibitor for depression. Low- impact aerobic exercise will prevent muscle atrophy without increasing pain at the knees. Relaxation can help decrease the patient's stress and anxiety. Long-acting opioids are generally avoided unless pain cannot be relieved by other medications.

What accurately describes passive acquired immunity (select all that apply)? a. Pooled gamma globulin b. Immunization with antigen c. Temporary for several months d. Immediate, lasting several weeks e. Maternal immunoglobulins in neonate f. Boosters may be needed for extended protection

Answer: A, C, D Passive acquired immunity is received from the injection of gamma globulin, provides immediate immunity, and may last for several weeks or months. Immunization with an antigen and the need for boosters contribute to active acquired immunity. Maternal immunoglobulins in the neonate provide temporary passive innate immunity.

The nurse is discharging a patient admitted with a transient ischemic attack (TIA). For which medications might the nurse expect to provide discharge instructions (select all that apply)? A. Clopidogrel (Plavix) B. Enoxaparin (Lovenox) C. Dipyridamole (Persantine) D. Enteric-coated aspirin (Ecotrin) E. Tissue plasminogen activator (tPA)

Answer: A, C, D Clopidogrel (Plavix), Dipyridamole (Persantine), & Enteric-coated aspirin (Ecotrin) Rationale: Aspirin is the most frequently used antiplatelet agent. Other drugs to prevent clot formation include clopidogrel (Plavix), dipyridamole (Persantine), ticlopidine (Ticlid), combined dipyridamole and aspirin (Aggrenox), and anticoagulant drugs, such as oral warfarin (Coumadin). Tissue plasminogen activator is a fibrinolytic medication used to treat ischemic stroke not prevent TIAs or strokes.

Benzodiazepines are indicated in the treatment of cases of delirium that have which cause? A. Polypharmacy B. Cerebral hypoxia C. Alcohol withdrawal D. Electrolyte imbalances

Answer: Alcohol withdrawal Rationale: Benzodiazepines can be used to treat delirium associated with sedative and alcohol withdrawal. However, these drugs may worsen delirium caused by other factors and must be used cautiously. Polypharmacy, cerebral hypoxia, and electrolyte imbalances are not treated with benzodiazepines.

While caring for a patient who is experience alcohol withdrawal, the nurse should (select all that apply) A. Monitor neurologic status on a routine basis. B. Provide a quiet, non-stimulating, dimly lit environment. C. Pad the side rails and place suction equipment at the bedside. D. Orient the patient to environment and the person with each contact. E. Administer anti-seizure drugs and sedatives to relieve symptoms during withdrawal.

Answer: A, C, D, E Monitor neurologic status on a routine basis, pad the side rails and place suction equipment at the bedside, Orient the patient to environment and the person with each contact, Administer anti-seizure drugs and sedatives to relieve symptoms during withdrawal. Rationale: For patients in withdrawal from all abused substances, nursing management includes monitoring neurologic status and vital signs and administering medications to prevent the progression of symptoms and increase patient comfort. Maintaining a well-lit environment that reduces sharp contrasts and shadows is important to reduce external stimuli. To prevent injury associated with seizure activity, nurses should keep suction equipment, an Ambu bag, and an oral or nasopharyngeal airway at the patient's bedside and use padded side rails.

Which immunoglobulins will initially protect a newborn baby of a breastfeeding mother (select all that apply)? a. IgA b. IgD c. IgE d. IgG e. IgM

Answer: A, D IgA is pasesd to the neonate in the colostrum and IgG crosses the placenta for fetal protection.

Common psychosocial reactions of the stroke patient to the stroke include (select all that apply) A. Depression B. Dissassociation C. Intellectualizaiton D. Sleep disturbances E. Denial of severity of stroke.

Answer: A, D, E Rationale: The patient with a stroke may experience many losses, including sensory, intellectual, communicative, functional, role behavior, emotional, social, and vocational losses. Some patients experience long-term depression, manifesting symptoms such as anxiety, weight loss, fatigue, poor appetite, and sleep disturbances. The time and energy required to perform previously simple tasks can result in anger and frustration. Frustration and depression are common in the first year after a stroke. A stroke is usually a sudden, extremely stressful event for the patient, caregiver, family, and significant others. The family is often affected emotionally, socially, and financially as their roles and responsibilities change. Reactions vary considerably but may involve fear, apprehension, denial of the severity of stroke, depression, anger, and sorrow.

During assessment of the patient with trigeminal neuralgia, the nurse should (select all that apply) A. Inspect all aspects of the mouth and teeth B. Assess the gag reflex and respiratory rate and depth C. Lightly palpate the affected side of the face for edema. D. Test for temperature and sensation perception on the face. E. Ask the patient to describe factors that initiate an episode.

Answer: A, D, E Inspect all aspects of the mouth and teeth, Test for temperature and sensation perception on the face, & Ask the patient to describe factors that initiate and episode. Rationale: Assessment of the attacks, including the triggering factors, characteristics, frequency, and pain management techniques, helps the nurse plan for patient care. Painful episodes are usually triggered by light cutaneous stimulation at a specific point (i.e., trigger zone) along the distribution of the nerve branches. Precipitating stimuli include chewing, tooth brushing, a hot or cold blast of air on the face, washing the face, yawning, or talking. Touch and tickle seem to predominate as causative triggers, rather than pain or changes in ambient temperature. The nursing assessment should include the patient's nutritional status, hygiene (especially oral), and behavior (including withdrawal). As a result of the attacks, the patient may eat improperly, neglect hygienic practices, wear a cloth over the face, and withdraw from interaction with others.

A nurse assesses a 38-year-old patient with joint pain and stiffness who was diagnosed with Stage III rheumatoid arthritis (RA). What characteristics should the nurse expect to observe (select all that apply)? A. Nodules present B. Consistent muscle strength C. Localized disease symptoms D. No destructive changes on x-ray E. Subluxation of joints without fibrous ankylosis

Answer: A, E Nodules present, Subluxation of joints without fibrous ankylosis Rationale: In Stage III severe RA, there may be extraarticular soft tissue lesions or nodules present, and there is subluxation without fibrous or bony ankylosis. The muscle strength is decreased because there is extensive muscle atrophy. The manifestations are systemic not localized. There is x-ray evidence of cartilage and bone destruction in addition to osteoporosis.

A healthy 65-year-old man who lives at home is at the clinic requesting a "flu shot." When assessing the patient, what other vaccinations should the nurse ask the patient about receiving (select all that apply)? A. Shingles B. Pneumonia C. Meningococcal D. Haemophilus influenzae type b (Hib) E. Measles, mumps, and rubella (MMR)

Answer: A,B, Rationale: The patient should receive the shingles (heres zoster) vaccine, Pneumovax, and influenza. The other options do not apply to this patient. Meningococcal vaccination is recommended for adults at risk (e.g., adults with anatomic or functional asplenia or persistent complement component deficiencies). Adults born before 1957 are generally considered immune to measles and mumps. Haemophilus influenzae type b (Hib) vaccination is only considered for adults with selected conditions (e.g., sickle cell disease, leukemia, HIV infection or for those who have anatomic or functional asplenia) if they have not been previously vaccinated.

A result of stimulation of the parasympathetic nervous system is (select all that apply) A. Constriction of the bronchi B. Dilation of skin blood vessels. C. Increased secretion of insulin D. Increased blood glucose levels. E. Relaxation of the urinary sphincters.

Answer: A,B,C,E Constriction of the bronchi, Dilation of skin blood vessels, Increased secretion of insulin, & Relaxation of the urinary sphincters. Rationale: Stimulation of the parasympathetic nervous system results in constriction of the bronchi, dilation of blood vessels to the skin, increased secretion of insulin, and relaxation of the urinary sphincter. Stimulation of the sympathetic nervous system results in increased blood glucose levels.

When caring for a patient with a known latex allergy, the nurse would monitor the patient closely for a cross-sensitivity to which foods (select all that apply)? A. Grapes B. Oranges C. Bananas D. Potatoes E. Tomatoes

Answer: A,C,D,E Grapes, Bananas, Potatoes, Tomatoes Rationale: Because some proteins in rubber are similar to food proteins, some foods may cause an allergic reaction in people who are allergic to latex. The most common of these foods are bananas, avocados, chestnuts, kiwi fruit, tomatoes, water chestnuts, guava, hazelnuts, potatoes, peaches, grapes, and apricots.

Where and into what do activated B lymphocytes differentiate? a. Spleen; natural killer cells that destroy infected cells b. Bone marrow; plasma cells that secrete immunoglobulins c. Thymus; memory B-cells that retain a memory of the antigen d. Bursa of Fabricius; helper cells that in turn activate additional B lymphocytes

Answer: B B lymphocytes activated in the bone marrow by the presentation of an antigen differentiate into many plasma cells that secrete immunoglobulins and only a few memory cells that retain recognition of teh antigen as foreign. Helper cells are T lymphocytes and natural killer cells are large, granular lymphocytes that are neither B no T lymphocytes. The spleen filters foreign substances from the blood. The thymus differentiates T lymphocytes. The bursa of Fabricius is found in birds, not humans.

What is included in the humoral immune response? a. Surveillance for malignant cell changes b. Production of antigen-specific immunoglobulins c. Direct attack of antigens by activated B lymphocytes d. Releasing cytokines responsible for destruction of antigens

Answer: B Production of immunoglobulins (or antibodies) by B lymphocytes differentiated into plasma cells is the essential component in humoral immunity. Tumor surveillance and the production of cytokines are function of T lymphocytes in cellular immunity and B lymphocytes do not directly attack antigens.

Creutzfeldt-Jakob disease is characterized by A. Remissions and exacerbations over many years. B. Memory impairment, muscle jerks, and blindness C. Parkinsonian symptoms, including muscle rigidity and tremors at rest. D. Increased intracranial pressure secondary to decreased CSF drainage.

Answer: B Rationale: Creutzfeldt-Jakob disease (CJD) is a fatal brain disorder caused by a prion protein. The earliest symptom of the disease may be memory impairment and behavioral changes. The disease progresses rapidly, with mental deterioration, involuntary movements (i.e., muscle jerks), weakness in the limbs, blindness, and eventually coma.

Decerebrate posture is documented in the chart of the patient that the nurse will be caring for. The nurse should know that the patient may have elevated ICP causing serious disruption of motor fibers in the midbrain and brainstem and will expect the patient's posture to look like which posture represented below? (flip side to see positions and answer) A. A. B. B. C. C. D. D.

Answer: B Rationale: Decerebrate posture is all four extremities in rigid extension with hyperpronation of the forearms and plantar flexion of feet. Decorticate posture is internal rotation and adduction of the arms with flexion of the elbows, wrists, and fingers from interruption of voluntary motor tracts in the cerebral cortex. Decorticate response on one side of the body and decerebrate response on the other side of the body may occur depending on the damage to the brain. Opisthotonic posture is decerebrate posture with the neck and back arched posteriorly and may be seen with traumatic brain injury.

For a patient who is suspected of having a stroke, one of the most important pieces of information that the nurse can obtain is A. time of the patient's last meal B. Time at which stroke symptoms first appeared. C. Patients hypertension history and management D. Family history of stroke and other cardiovascular disease

Answer: B Rationale: During initial evaluation, the most important point in the patient's history is the time since onset of stroke symptoms. If the stroke is ischemic, recombinant tissue plasminogen activator (tPA) must be administered within 3 to 4.5 hours of the onset of clinical signs of ischemic stroke; tPA reestablishes blood flow through a blocked artery and prevents brain cell death in patients with acute onset of ischemic stroke.

A 58-year-old man who is waiting for a kidney transplant asks the nurse to explain the difference between a negative and positive cross match. Which statement by the nurse would be the most accurate response? A. "A negative cross match means that both the donor and recipient are Rh negative, and the transplant is safe." B. "A negative cross match means that no preformed antibodies are present and the transplant would be safe." C. "A positive cross match means the blood type is the same between donor and recipient, and the transplant is safe." D. "A positive cross match means that both the donor and the recipient have antigens that are similar, and the transplant would be safe."

Answer: B "A negative cross match means that no preformed antibodies are present and the transplant would be safe." Rationale: A cross match uses serum from the recipient mixed with donor lymphocytes to test for any preformed antibodies to the potential donor organ. A positive cross match indicates that the recipient has cytotoxic antibodies to the donor and is an absolute contraindication to transplantation. A negative cross match indicates that no preformed antibodies are present and it is safe to proceed with transplantation.

A 66-year-old man with type 2 diabetes mellitus and atrial fibrillation has begun taking glucosamine and chondroitin for osteoarthritis. Which question is most important for the nurse to ask? A. "Did you have any hypoglycemic reactions?" B. "Have you noticed any bruising or bleeding?" C. "Have you had any dizzy spells when standing up?" D. "Do you have any numbness or tingling in your feet?"

Answer: B "Have you noticed any bruising or bleeding?" Rationale:Glucosamine and chondroitin are dietary supplements commonly used to treat osteoarthritis. Both glucosamine and chondroitin may increase the risk of bleeding. Anticoagulant therapy is indicated for patients with atrial fibrillation to reduce the risk of a thromboembolism and a stroke. Use of glucosamine and chondroitin along with an anticoagulant may precipitate excessive bleeding. Glucosamine may decrease the effectiveness of insulin or other drugs used to control blood glucose levels, and hyperglycemia may occur. Peripheral neuropathy symptoms that can develop with prolonged hyperglycemia include numbness and tingling in the feet.

The nurse provides information to the caregiver of a 68-year-old man with epilepsy who has tonic-clonic seizures. Which statement, if made by the caregiver, requires further teaching? A. "It is normal for a person to be sleepy after a seizure." B. "I should call 911 if breathing stops during the seizure." C. "The jerking movements may last for 30 to 40 seconds." D. "Objects should not be placed in the mouth during a seizure."

Answer: B "I should call 911 if breathing stops during the seizure." Rationale:Caregivers do not need to call an ambulance or send a person to the hospital after a single seizure unless the seizure is prolonged, another seizure immediately follows, or extensive injury has occurred. Altered breathing is a clinical manifestation of a tonic-clonic seizure. Contact emergency medical services (or call 911) if breathing stops for more than 30 seconds. No objects (e.g., oral airway, padded tongue blade) should be placed in the mouth. Lethargy is common in the postictal phase of a seizure. Jerking of the extremities occurs during the clonic phase of a tonic-clonic seizure. The clonic phase may last 30 to 40 seconds.

When reinforcing health teaching about the management of osteoarthritis (OA), the nurse determines that the patient needs additional instruction after making which statement? A. "I should take the Naprosyn as prescribed to help control the pain." B. "I should try to stay standing all day to keep my joints from becoming stiff." C. "I can use a cane if I find it helpful in relieving the pressure on my back and hip." D. "A warm shower in the morning will help relieve the stiffness I have when I get up."

Answer: B "I should try to stay standing all day to keep my joints from becoming stiff." Rationale: It is important to maintain a balance between rest and activity to prevent overstressing the joints with OA. Naproxen (Naprosyn) may be used for moderate to severe OA pain. Using a cane and warm shower to help relieve pain and morning stiffness are helpful.

The nurse is providing medication teaching for a 52-year-old man who is taking clonidine (Catapres) to control cravings for nicotine as part of a smoking cessation program. Which statement, if made by the patient, indicates an understanding of the adverse effects of this medication? A. "I will limit intake of high-fiber foods to prevent diarrhea." B. "I will get up slowly to prevent dizziness or fainting spells." C. "I may experience an altered sense of taste, especially with salty foods." D. "I will be safer while driving because the medication improves alertness."

Answer: B "I will get up slowly to prevent dizziness or fainting spells." Rationale: The most common adverse effects reported by patients taking clonidine are dry mouth, drowsiness, dizziness, constipation, and sedation.

The nurse cares for a 63-year-old woman taking prednisone (Deltasone) and acyclovir (Zovirax) for Bell's palsy. It is most important for the nurse to follow up on which patient statement? A. "I can take both medications with food or milk." B. "I will take these medications for 2 to 3 months." C. "I can take acetaminophen with the prescribed medications." D. "Chances of a full recovery are good if I take the medications."

Answer: B "I will take these medications for 2 to 3 months." Rationale: Prednisone and acyclovir will usually be prescribed for 10 days. Prednisone will be tapered over the last 4 days of treatment. Oral prednisone and acyclovir may be taken with food or milk to decrease gastrointestinal upset. Patients with Bell's palsy usually begin recovery in 2 to 3 weeks, and most patients have complete recovery in 2 to 3 months. Patients have the best chance for full recovery if prednisone is initiated before complete paralysis occurs. There are no serious drug interactions among prednisone, acyclovir, and acetaminophen.

The nurse teaches a 38-year-old woman who has migraine headaches about sumatriptan (Imitrex). Which statement by the patient requires an intervention by the nurse? A. "The injection might feel like a bee sting." B. "This medicine will prevent a migraine headache." C. "I can take another dose if the first does not work." D "This drug for migraine headaches could cause birth defects."

Answer: B "This medicine will prevent a migraine headache." Rationale: Sumatriptan is given to abort an ongoing migraine headache and is not used to prevent migraine headaches. When given as a subcutaneous injection, this drug may cause transient pain and redness at the injection site. This drug may be repeated after a specified time period if the first dose is not effective. This drug should be avoided during pregnancy and is classified as an FDA Pregnancy Risk Category C drug.

Which statement made by the nurse is most appropriate in teaching patient interventions to minimize the effects of seasonal allergic rhinitis? A. "You will need to get rid of your pets." B. "You should sleep in an air-conditioned room." C. "You would do best to stay indoors during the winter months." D. "You will need to dust your house with a dry feather duster twice a week."

Answer: B "You should sleep in an air-conditioned room." Rationale: Seasonal allergic rhinitis is most commonly caused by pollens from trees, weeds, and grasses. Airborne allergies can be controlled by sleeping in an air-conditioned room, daily damp dusting, covering the mattress and pillows with hypoallergenic covers, and wearing a mask outdoors.

When admitting a patient, the nurse must assess the patient for substance use based on the knowledge that long-term use of addictive substances leads to A. Development of coexisting psychiatric illnesses. B. A higher risk for complications from underlying health problems. C. Increased availability of dopamine, resulting in decreased sleep requirements. D. Potentiation of effects of similar drugs taken when the individual is drug free.

Answer: B A higher risk for complications from underlying health problems. Rationale: Almost every drug of abuse harms some tissue or organ of the body.

The nurse is providing care for a patient who has been admitted with alcohol withdrawal delirium. Which intervention should be the first priority for the nurse? A. Applying physical restraints B. Administering benzodiazepines C. Questioning the patient about his last alcohol intake D. Providing information on community resources for alcohol treatment

Answer: B Administering benzodiazepines Rationale: Benzodiazepines are frequently used to treat the signs and symptoms of alcohol withdrawal delirium and should be administered as soon as possible. Knowledge about the patient's last alcohol intake is not necessary since withdrawal symptoms are already occurring. Physical restraints are rarely necessary if drugs administered during withdrawal are appropriately used. Information about treatment for alcohol dependency is not appropriate at this acute stage.

A 59-year-old female patient, who has frontotemporal lobar degeneration, has difficulty with verbal expression. One day she walks out of the house and goes to the gas station to get a soda but does not understand that she needs to pay for it. What is the best thing the nurse can suggest to this patient's husband to keep the patient safe during the day while the husband is at work? A. Assisted living B. Adult day care C. Advance directives D. Monitor for behavioral changes

Answer: B Adult day care Rationale: To keep this patient safe during the day while the husband is at work, an adult day care facility would be the best choice. This patient would not need assisted living. Advance directives are important but are not related to her safety. Monitoring for behavioral changes will not keep her safe during the day.

The nurse would suspect cocaine overdose in the patient who is experiencing A. Craving, restlessness, and irritability. B. Agitation, cardiac dysrhythmias, and seizures. C. Diarrhea, nausea and vomiting, and confusion. D. Slow, shallow respirations; hyporeflexia; and blurred vision.

Answer: B Agitation, cardiac dysrhythmias, and seizures. Rationale: The following symptoms may occur with a cocaine overdose: agitation; increases in temperature, pulse, respiratory rate, and blood pressure; cardiac dysrhythmias and myocardial infarction; hallucinations; seizures; and possible death.

A female patient has left-sided hemiplegia following an ischemic stroke that she experienced 4 days earlier. How should the nurse best promote the health of the patient's integumentary system? A. Position the patient on her weak side the majority of the time. B. Alternate the patient's positioning between supine and side-lying. C. Avoid the use of pillows in order to promote independence in positioning. D. Establish a schedule for the massage of areas where skin breakdown emerges.

Answer: B Alternate the patient's positioning between supine and side-lying. A position change schedule should be established for stroke patients. An example is side-back-side, with a maximum duration of 2 hours for any position. The patient should be positioned on the weak or paralyzed side for only 30 minutes. Pillows may be used to facilitate positioning. Areas of skin breakdown should never be massaged.

When assessing the accessory nerve, what should the nurse do? A. Assess the gag reflex by stroking the posterior pharynx. B. Ask the patient to shrug the shoulders against resistance. C. Ask the patient to push the tongue to either side against resistance. D. Have the patient say "ah" while visualizing elevation of soft palate.

Answer: B Ask the patient to shrug the shoulders against resistance. Rationale:

How should the nurse most accurately assess the position sense of a patient with a recent traumatic brain injury? A. Ask the patient to close his or her eyes and slowly bring the tips of the index fingers together. B. Ask the patient to stand with the feet together and eyes closed and observe for balance maintenance. C. Ask the patient to close his or her eyes and identify the presence of a common object on the forearm. D. Place the two points of a calibrated compass on the tips of the fingers and toes and ask the patient to discriminate the points.

Answer: B Ask the patient to stand with the feet together and eyes closed and observe for balance maintenance. Rationale: The Romberg test is an assessment of position sense in which the patient stands with the feet together and then closes his or her eyes while attempting to maintain balance. The other cited tests of neurologic function do not directly assess position sense.

The most common early symptom of a spinal cord tumor is A. urinary incontinence. B. Back pain that worsens with activity C. Paralysis below the level of involvement. D. Impaired sensation of pain, temperature, and light touch

Answer: B Back pain the worsens with activity Rationale: The most common early symptom of a spinal cord tumor outside the cord is pain in the back, with radicular pain simulating intercostal neuralgia, angina, or herpes zoster. The location of the pain depends on the level of compression. The pain worsens with activity, coughing, straining, and lying down.

The nurse is caring for a patient admitted with a subdural hematoma following a motor vehicle accident. Which change in vital signs would the nurse interpret as a manifestation of increased intracranial pressure (ICP)? A. Tachypnea B. Bradycardia C. Hypotension D. Narrowing pulse pressure

Answer: B Bradycardia Rationale: Bradycardia could indicate increased ICP. Changes in vital signs (known as Cushing's triad) occur with increased ICP. They consist of increasing systolic pressure with a widening pulse pressure, bradycardia with a full and bounding pulse, and irregular respirations.

The nurse is caring for a patient newly diagnosed with HIV. The patient asks what would determine the actual development of AIDS. The nurse's response is based on the knowledge that what is a diagnostic criterion for AIDS? A. Presence of HIV antibodies B. CD4+ T cell count below 200/µL C. Presence of oral hairy leukoplakia D. White blood cell count below 5000/µL

Answer: B CD4+ T cell count below 200/µ Rationale: Diagnostic criteria for AIDS include a CD4+ T cell count below 200/µL and/or the development of specified opportunistic infections, cancers, wasting syndrome, or dementia. The other options may be found in patients with HIV disease but do not define the advancement of HIV infection to AIDS.

Which statement(s) accurately describe(s) mild cognitive impairment (select all that apply)? A. Always progresses to AD B. Caused by variety of factors and may progress to AD C. Should be aggressively treated with acetycholinesterase drugs D. Caused by vascular infarcts that, if treated, will delay progression to AD E. Patient is usually not aware that there is a problem with his or her memory

Answer: B Caused by variety of factors and may progress to AD Rationale: Although some individuals with mild cognitive impairment (MCI) revert to normal cognitive function or do not go on to develop Alzheimer's disease (AD), those with MCI are at high risk for AD. No drugs have been approved for the treatment of MCI. A person with MCI is often aware of a significant change in memory.

Because the incidence of Lyme disease is very high in Wisconsin, the public health nurse is planning to provide community education to increase the number of people who seek health care promptly after a tick bite. What information should the nurse provide when teaching people who are at risk for a tick bite? A. The best therapy for the acute illness is an IV antibiotic. B. Check for an enlarging reddened area with a clear center. C. Surveillance is necessary during the summer months only. D. Antibiotics will prevent Lyme disease if taken for 10 da

Answer: B Check for an enlarging reddened area with a clear center. Rationale: Following a tick bite, the expanding "bull's eye rash" is the most characteristic symptom that usually occurs in 3 to 30 days. There may also be flu-like symptoms and migrating joint and muscle pain. Active lesions are treated with oral antibiotics for 2 to 3 weeks, and doxycycline is effective in preventing Lyme disease when given within 3 days after the bite of a deer tick. IV therapy is used with neurologic or cardiac complications. Although ticks are most prevalent during summer months, residents of high-risk areas should check for ticks whenever they are outdoors.

The nurse was accidentally stuck with a needle used on an HIV-positive patient. After reporting this, what care should this nurse first receive? A. Personal protective equipment B. Combination antiretroviral therapy C. Counseling to report blood exposures D. A negative evaluation by the manager

Answer: B Combination antiretroviral therapy Rationale: Postexposure prophylaxis with combination antiretroviral therapy can significantly decrease the risk of infection. Personal protective equipment should be available although it may not have stopped this needle stick. The needle stick has been reported. The negative evaluation may or may not be needed but would not occur first.

A nursing measure that is indicated to reduce the potential for seizures and increased intracranial pressure in the patient with bacterial meningitis is A. Administering codeine for relief of head and neck pain. B. Controlling fever with prescribed rugs and cooling techniques C. Keeping the room dark and quiet to minimize environmental stimulation. D. Maintaining the patient on strict bed rest with the head of the bed slightly elevated.

Answer: B Controlling fever with prescribed rugs and cooling techniques Rationale: Fever must be vigorously managed because it increases cerebral edema and the frequency of seizures. Neurologic damage may result from an extremely high temperature over a prolonged period. Acetaminophen or aspirin may be used to reduce fever; other measures, such as a cooling blanket or tepid sponge baths with water, may be effective in lowering the temperature.

Antiretroviral drugs are used to A. Cure acute HIV infection B. Decrease viral RNA levels. C. Treat opportunistic diseases. D. Decrease pain and symptoms in terminal disease

Answer: B Decrease viral RNA levels. Rationale: The goals of drug therapy in HIV infection are to (1) decrease the viral load, (2) maintain or raise CD4+ T cell counts, and (3) delay onset of HIV infection-related symptoms and opportunistic diseases.

A nurse is working with a 73-year-old patient with osteoarthritis (OA). In assessing the patient's understanding of this disorder, the nurse concludes teaching has been effective when the patient uses which description of the condition? A. Joint destruction caused by an autoimmune process B. Degeneration of articular cartilage in synovial joints C. Overproduction of synovial fluid resulting in joint destruction D. Breakdown of tissue in non-weight-bearing joints by enzymes

Answer: B Degeneration of articular cartilage in synovial joints Rationale: OA is a degeneration of the articular cartilage in diarthrodial (synovial) joints from damage to the cartilage. The condition has also been referred to as degenerative joint disease. OA is not an autoimmune disease. There is no overproduction of synovial fluid causing destruction or breakdown of tissue by enzymes.

The nurse observes a 74-year-old man with Parkinson's disease rocking side to side while sitting in the chair. Which action by the nurse is most appropriate? A. Provide the patient with diversional activities. B. Document the activity in the patient's health record. C. Take the patient's blood pressure sitting and standing. D. Ask if the patient is feeling either anxious or depressed.

Answer: B Document the activity in the patient's health record Rationale:

A nurse pans care for the patient with increased intracranial pressure with the knowledge that the best way to position the patient is to A. Keep the head of the bed flat B. Elevate the head of the bed to 30 degrees C. Maintain patient on the left side with the head supported on a pillow D. Use a continuous-rotation bed to continuously change patient position.

Answer: B Elevate the head of the bed to 30 degrees Rationale: The nurse should maintain the patient with abnormal ICP in the head-up position. Elevation of the head of the bed to 30 degrees enhances respiratory exchange and aids in decreasing cerebral edema. The nurse should position the patient to prevent extreme neck flexion, which can cause venous obstruction and contribute to elevation in ICP. Elevation of the head of the bed also reduces sagittal sinus pressure, promotes drainage from the head through the valveless venous system and jugular veins, and decreases the vascular congestion that can produce cerebral edema. However, raising the head of the bed above 30 degrees may decrease the cerebral perfusion pressure (CPP) by lowering systemic blood pressure. The effects of elevation of the head of the bed on the ICP and CPP must be evaluated carefully

When administering medications to the patient with gout, the nurse would recognize that which drug is used as a treatment for this disease? A. Colchicine B. Febuxostat C. Sulfasalazine D. Cyclosporine

Answer: B Febuxostat Rationale: Febuxostat (Uloric), a selective inhibitor of xanthine oxidase, is administered for long-term management of hyperuricemia in persons with chronic gout. An acute episode of gout is treated with colchicine and nonsteroidal antiinflammatory drugs (NSAIDs).

A hospital has seen a recent increase in the incidence of hospital care-associated infections (HAIs). Which measure should be prioritized in the response to this trend? A. Use of gloves during patient contact B. Frequent and thorough hand washing C. Prophylactic, broad-spectrum antibiotics D. Fitting and appropriate use of N95 masks

Answer: B Frequent and thorough hand washing Rationale: Hand washing remains the mainstay of the prevention of HAIs. Gloves, masks, and antibiotics may be appropriate in specific circumstances, but none of these replaces the central role of vigilant, thorough hand washing between patients and when moving from one task to another, even with the same patient.

The nurse is teaching a group of young adults who live in a dormitory about the prevention of antibiotic-resistant infections. What should be included in the teaching plan? A. Save leftover antibiotics for future uses. B. Hand washing can prevent many infections. C. Antibiotics are indicated for preventing most colds. D. Stop taking prescribed antibiotics when symptoms improve.

Answer: B Hand washing can prevent many infections Rationale: Hand washing is the single most important action to prevent infections. Antibiotics are used to treat bacterial infections, not viral colds and flu. Patients should complete the entire prescription of antibiotics to prevent the development of resistant bacteria. Antibiotics should not be taken to prevent infections unless they are given prophylactically before undergoing certain surgeries and dental work.

The nurse would expect to find what clinical manifestation in a patient admitted with a left-sided stroke? A. Impulsivity B. Impaired speech C. Left-side neglect D. Short attention span

Answer: B Impaired speech Rationale: Clinical manifestations of left-sided brain damage include right hemiplegia, impaired speech/language, impaired right/left discrimination, and slow and cautious performance. Impulsivity, left-sided neglect, and short attention span are all manifestations of right-sided brain damage.

In assessing the joints of a patient with osteoarthrits, the nurse understands that Heberden's nodes A. Are often red, swollen, and tender B. Indicate osteophyte formation at the DIP joints. C. Are the result of pannus formation at the PIP joints. D. Occur form deterioration of cartilage by proteolytic enzymes.

Answer: B Indicate osteophyte formation at the DIP joints. Rationale: Heberden's nodes are bony deformities in the distal interphalangeal joints that are indicative of osteophyte formation and loss of joint space in osteoarthritis.

The physician orders alteplase (Activase) for a 58-year-old man diagnosed with an acute ischemic stroke. Which nursing action is most appropriate? A. Administer the medication by an IV route at 15 mL/hr for 24 hours. B. Insert two or three large-bore IV catheters before administering the medication. C. If gingival bleeding occurs, discontinue the medication and notify the physician. D. Reduce the medication infusion rate for a systolic blood pressure above 180 mm Hg.

Answer: B Insert two or three large-bore IV catheters before administering the medication. Rationale: Before giving alteplase, the nurse should start two or three large bore IVs. Bleeding is a major complication with fibrinolytic therapy, and venipunctures should not be attempted after alteplase is administered. Altepase is administered IV with an initial bolus dose followed by an infusion of the remaining medication within the next 60 minutes. Gingival bleeding is a minor complication and may be controlled with pressure or ice packs. Control of blood pressure is critical prior to altepase administration and for the following 24 hours. Before administering altepase, a systolic pressure above 180 mm Hg or diastolic pressure above 110 mm Hg requires aggressive blood pressure treatment to reduce the risk of cerebral hemorrhage.

10. The nurse is caring for a patient with peripheral neuropathy who is going to have EMG studies tomorrow morning. The nurse should A. Ensure the patient has an empty bladder B. Instruct the patient that there is no risk of electric shock C. Ensure the patient has no metallic jewelry or metal fragments D. Instruct the patient that she or he may experience pain during the study.

Answer: B Instruct the patient that there is no risk of electric shock Rationale: Electromyography (EMG) is used to assess electrical activity associated with nerves and skeletal muscles. Activity is recorded by insertion of needle electrodes to detect muscle and peripheral nerve disease. The nurse should inform the patient that pain and discomfort are associated with insertion of needles. There is no risk of electric shock with this procedure.

Substance abuse problems in older adults are usually related to A. Use of drugs and alcohol as a social activity. B. Misuse of prescribed and over-the-counter drugs and alcohol. C. Continuing the use of illegal drugs initiated during middle age. D. A pattern of binge drinking for weeks or months with periods of sobriety.

Answer: B Misuse of prescribed and over-the-counter drugs and alcohol. Rationale: Health care providers are much less likely to recognize substance abuse and dependence in older adults than in younger adults. Older adults may misuse prescription and over-the-counter drugs and alcohol.

A patient admitted to the hospital early in the morning for elective surgery asks the nurse if he can go outdoors to have one last cigarette before surgery. What is the nurse's most appropriate response to the patient's request? A. Tell the patient that smoking is not allowed anywhere on hospital property. B. Notify the physician that the patient may need an over-the-counter nicotine replacement agent. C. Inform him that this would be an ideal time for him to quit smoking because he won't want to smoke right after surgery. D. Contact the anesthesia care provider to ensure that the patient's smoking history is noted as a risk factor for surgery.

Answer: B Notify the physician that the patient may need an over-the-counter nicotine replacement agent. Rationale: When smokers are hospitalized, it is important to acknowledge their addiction and encourage them to stop tobacco use. In a stressful time such as surgery, nicotine replacement agents are helpful to control withdrawal cravings and should be used with any support and assistance needed. Although smoking is a risk for postoperative complications, the anesthesia care provider would assess and manage the effects of tobacco use. Just telling the patient that smoking is not allowed is not therapeutic.

The nurse is admitting a patient who is scheduled for knee arthroscopy related to osteoarthritis (OA). Which finding should the nurse expect to be present on examination of the patient's knees? A. Ulnar drift B. Pain with joint movement C. Reddened, swollen affected joints D. Stiffness that increases with movement

Answer: B Pain with joint movement Rationale: OA is characterized predominantly by joint pain upon movement and is a classic feature of the disease. Ulnar drift occurs with rheumatoid arthritis (RA) not osteoarthritis. Not all joints are reddened or swollen. Only Heberden's and Bouchard's nodes may be. Stiffness decreases with movement.

The nurse in the long-term care facility cares for a 70-year-old man with severe (late-stage) dementia who is undernourished and has problems chewing and swallowing. What should the nurse include in the plan of care for this patient? A. Turn on the television to provide a distraction during meals. B. Provide thickened fluids and moist foods in bite-size pieces. C. Limit fluid intake during scheduled meals to prevent aspiration. D. Allow the patient to select favorite foods from the menu choices.

Answer: B Provide thickened fluids and moist foods in bite-size pieces. Rationale: If patients with dementia have problems chewing or swallowing, pureed foods, thickened liquids, and nutritional supplements should be provided. Foods that are easy to swallow are moist and should be in bite-size pieces. Distractions at mealtimes, including the television, should be avoided. Fluids should not be limited but offered frequently; fluids should be thickened. Patients with severe (late-stage) dementia have difficulty understanding words and would not have the cognitive ability to select menu choices.

A 50-year-old male patient has been diagnosed with amyotrophic lateral sclerosis (ALS). What nursing intervention is most important to help prevent a common cause of death for patients with ALS? A. Reduce fat intake. B. Reduce the risk of aspiration. C. Decrease injury related to falls. D. Decrease pain secondary to muscle weakness.

Answer: B Reduce the risk of aspiration. Rationale: Reducing the risk of aspiration can help prevent respiratory infections that are a common cause of death from deteriorating muscle function. Reducing fat intake may reduce cardiovascular disease, but this is not a common cause of death for patients with ALS. Decreasing injury related to falls and decreasing pain secondary to muscle weakness are important nursing interventions for patients with ALS but are unrelated to causes of death for these patients.

A 68-year-old man with suspected bacterial meningitis has just had a lumbar puncture in which cerebrospinal fluid was obtained for culture. Which medication should the nurse administer first? A. Codeine B. Phenytoin (Dilantin) C. Ceftriaxone (Rocephin) D. Acetaminophen (Tylenol)

Answer: C Ceftriaxone (Rocephin) Rationale: Bacterial meningitis is a medical emergency. When meningitis is suspected, antibiotic therapy (e.g., ceftriaxone) is instituted immediately after the collection of specimens for cultures, and even before the diagnosis is confirmed. Dexamethasone may also be prescribed before or with the first dose of antibiotics. The nurse should collaborate with the health care provider to manage the headache (with codeine), fever (with acetaminophen), and seizures (with phenytoin).

The nurse is caring for a patient admitted with a spinal cord injury following a motor vehicle accident. The patient exhibits a complete loss of motor, sensory, and reflex activity below the injury level. The nurse recognizes this condition as which of the following? A. Central cord syndrome B. Spinal shock syndrome C. Anterior cord syndrome D. Brown-Séquard syndrome

Answer: B Spinal shock syndrome Rationale: About 50% of people with acute spinal cord injury experience a temporary loss of reflexes, sensation, and motor activity that is known as spinal shock. Central cord syndrome is manifested by motor and sensory loss greater in the upper extremities than the lower extremities. Anterior cord syndrome results in motor and sensory loss but not reflexes. Brown-Séquard syndrome is characterized by ipsilateral loss of motor function and contralateral loss of sensory function.

Which characteristic of a patient's recent seizure is consistent with a focal seizure? A. The patient lost consciousness during the seizure. B. The seizure involved lip smacking and repetitive movements. C. The patient fell to the ground and became stiff for 20 seconds. D. The etiology of the seizure involved both sides of the patient's brain.

Answer: B The seizure involved lip smacking and repetitive movements. Rationale: The most common complex focal seizure involves lip smacking and automatisms (repetitive movements that may not be appropriate). Loss of consciousness, bilateral brain involvement, and a tonic phase are associated with generalized seizure activity

The mother does not want her child to have any extra immunizations for diseases that no longer occur. What teaching about immunization should the nurse provide this mother? A. There is currently no need for those older vaccines. B. There is a reemergence of some of the infections, such as pertussis. C. There is no longer an immunization available for some of those diseases. D. The only way to protect your child is to have the federally required vaccines.

Answer: B There is a reemergence of some of the infections, such as pertussis Rationale: Teaching the mother that some of the diseases are reemerging and the damage they can do to her child gives the mother the information to make an informed decision. The immunizations still exist and do protect individuals.

A patient's low hemoglobin and hematocrit have necessitated a transfusion of packed red blood cells (RBCs). Shortly after the first unit of RBCs starts to infuse, the patient develops signs and symptoms of a transfusion reaction. Which type of hypersensitivity reaction has the patient experienced? A. Type I B. Type II C. Type III D. Type IV

Answer: B Type II Rationale: Transfusion reactions are characterized as a type II (cytotoxic) reaction in which agglutination and cytolysis occur. Type I hypersensitivity reactions are IgE-mediated reactions to specific allergens (e.g., exogenous pollen, food, drugs, or dust). Type III reactions are immune-complex reactions that occur secondary to antigen-antibody complexes. Type IV reactions are delayed cell-mediated immune response reactions.

The patient has been diagnosed with the mild cognitive impairment stage of Alzheimer's disease. What nursing interventions should the nurse expect to use with this patient? A. Treat disruptive behavior with antipsychotic drugs. B . Use a calendar and family pictures as memory aids. C. Use a writing board to communicate with the patient. D. Use a wander guard mechanism to keep the patient in the area.

Answer: B Use a calendar and family pictures as memory aids. Rationale: The patient with mild cognitive impairment will have problems with memory, language, or another essential cognitive function that is severe enough to be noticeable to others but does not interfere with activities of daily living. A calendar and family pictures for memory aids will help this patient. This patient should not yet have disruptive behavior or get lost easily. Using a writing board will not help this patient with communication.

A 25-year-old male patient has been diagnosed with HIV. The patient does not want to take more than one antiretroviral drug. What reasons can the nurse tell the patient about for taking more than one drug? A. Together they will cure HIV. B. Viral replication will be inhibited. C. They will decrease CD4+ T cell counts. D. It will prevent interaction with other drugs.

Answer: B Viral replication will be inhibited Rationale: The major advantage of using several classes of antiretroviral drugs is that viral replication can be inhibited in several ways, making it more difficult for the virus to recover and decreasing the likelihood of drug resistance that is a major problem with monotherapy. Combination therapy also delays disease progression and decreases HIV symptoms and opportunistic diseases. HIV cannot be cured. CD4+ T cell counts increase with therapy. There are dangerous interactions with many antiretroviral drugs and other commonly used drugs.

A patient has been receiving scheduled doses of phenytoin (Dilantin) and begins to experience diplopia. The nurse immediately assesses the patient for A. an aura or focal seizure. B. nystagmus or confusion. C. abdominal pain or cramping. D. irregular pulse or palpitations.

Answer: B nystagmus or confusion. Rationale: Diplopia is a sign of phenytoin toxicity. The nurse should assess for other signs of toxicity, which include neurologic changes, such as nystagmus, ataxia, confusion, dizziness, or slurred speech. An aura, focal seizure, abdominal pain or cramping, irregular pulse, or palpitations are not associated with phenytoin toxicity.

The patient has vancomycin-resistant enterococci (VRE) infection in a surgical wound. What infection precautions should the nurse use to best prevent transmission of the infection to the nurse, other patients, staff, and those outside the hospital? A. Droplet precautions B. Contact precautions C. Airborne precautions D. Standard precautions

Answer: B Contact precautions Rationale: Contact precautions are used to minimize the spread of pathogens that are acquired from direct or indirect contact. Droplet precautions are used with pathogens that are spread through the air at close contact and that affect the respiratory system or mucous membranes (e.g., influenza, pertussis). Airborne precautions are used if the organism can cause infection over long distances when suspended in the air (e.g., TB, rubeola). Standard precautions are used with all patients and included in the transmission-based precautions above.

The early stage of AD is characterized by A. Non noticeable change in behavior. B. Memory problems and mild confusion. C. Increased time spent sleeping or in bed. D. Incontinence, agitation, and wandering behavior

Answer: B Memory problems and mild confusion. Rationale: An initial sign of AD is a subtle deterioration in memory.

Which statement about metabolic side effects of ART is true (select all that apply)? A. These are annoying symptoms that are ultimately harmless. B. ART- related boy changes include central fat accumulation and peripheral wasting. C. Lipid abnormalities include increases in triglycerides and decreases in high-density cholesterol. D. Insulin resistance and hyperlipidemia can be treated with drugs to control glucose and cholesterol. E. Compared to uninfected people, insulin resistance and hyperlipidemia are more difficult to treat in HIV-infected patients.

Answer: B, C, D, ART- related boy changes include central fat accumulation and peripheral wasting, Lipid abnormalities include increases in triglycerides and decreases in high-density cholesterol, & Insulin resistance and hyperlipidemia can be treated with drugs to control glucose and cholesterol. Rationale: Some HIV-infected patients, especially those who have been infected and have received ART for a long time, develop a set of metabolic disorders that include changes in body shape (e.g., fat deposits in the abdomen, upper back, and breasts along with fat loss in the arms, legs, and face) as a result of lipodystrophy, hyperlipidemia (i.e., elevated triglyceride levels and decreases in high-density lipoprotein levels), insulin resistance and hyperglycemia, bone disease (e.g., osteoporosis, osteopenia, avascular necrosis), lactic acidosis, and cardiovascular disease.

When providing community health care teaching regarding the early warning signs of Alzheimer's disease, which signs should the nurse advise family members to report (select all that apply)? A. Misplacing car keys B. Losing sense of time C. Difficulty performing familiar tasks D. Problems with performing basic calculations E/ Becoming lost in a usually familiar environment

Answer: B, C, D, E Losing sense of time, Difficulty performing familiar tasks, Problems with performing basic calculations, Becoming lost in a usually familiar environment Rationale: Difficulty performing familiar tasks, problems with performing basic calculations, losing sense of time, and becoming lost in a usually familiar environment are all part of the early warning signs of Alzheimer's disease. Misplacing car keys is a normal frustrating event for many people.

Which normal nervous system changes of aging put the geriatric person at higher risk of falls (select all that apply)? A. Memory deficit B. Sensory deficit C. Motor function deficit D. Cranial and spinal nerves E. Reticular activation system F. Central nervous system changes

Answer: B, C, F Sensory deficit, Motor function deficit, & Central nervous system changes Rationale: An older person is at a higher risk for falls because the changes in the nervous system decrease the sensory function that leads to poor ability to maintain balance and a widened gait. The motor function deficit decreases muscle strength and agility. The central nervous system changes in the brain lead to a diminished kinesthetic sense or position sense. Memory deficits, normal changes of cranial and spinal nerves, and the reticular activation system do not contribute to the increased risk of falls.

A 68-year-old patient with a spinal cord injury has a neurogenic bowel. Beyond the use of bisacodyl (Dulcolax) suppositories and digital stimulation, which measures should the nurse teach the patient and the caregiver to assist the patient with bowel evacuation (select all that apply)? A. Drink more milk. B. Eat 20-30 g of fiber per day. C. Use oral laxatives every day. D. Drink 1800 to 2800 mL of water or juice. E. Establish bowel evacuation time at bedtime.

Answer: B, D Rationale: The patient with a spinal cord injury and neurogenic bowel should eat 20-30 g of fiber and drink 1800 to 2800 mL of water or juice each day. Milk may cause constipation. Daily oral laxatives may cause diarrhea and are avoided unless necessary. Bowel evacuation time is usually established 30 minutes after the first meal of the day to take advantage of the gastrocolic reflex induced by eating.

The patient is admitted to the ED with fever, swollen lymph glands, sore throat, headache, malaise, joint pain, and diarrhea. What nursing measures will help identify the need for further assessment of the cause of this patient's manifestations (select all that apply)? A. Assessment of lung sounds B. Assessment of sexual behavior C. Assessment of living conditions D. Assessment of drug and syringe use E. Assessment of exposure to an ill person

Answer: B, D Assessment of sexual behavior & Assessment of drug and syringe use Rationale: With these symptoms, assessing this patient's sexual behavior and possible exposure to shared drug equipment will identify if further assessment for the HIV virus should be made or the manifestations are from some other illness (e.g., lung sounds and living conditions may indicate further testing for TB).

The nurse is providing care for a patient who has been admitted to the hospital with a head injury and who requires regular neurologic and vital sign assessment. Which assessments will be components of the patient's score on the Glasgow Coma Scale (GCS) (select all that apply)? A. Judgment B. Eye opening C. Abstract reasoning D. Best verbal response E. Best motor response F. Cranial nerve function

Answer: B, D, E Eye opening, Best verbal response, & Best motor response Rationale: The three dimensions of the GCS are eye opening, best verbal response, and best motor response. Judgment, abstract reasoning, and cranial nerve function are not components of the GCS.

When establishing a diagnosis of MS, the nurse should teach the patient about what diagnostic studies (select all that apply)? A. EEG B. CT scan C. Carotid duplex scan D. Evoked response testing E. Cerebrospinal fluid analysis

Answer: B, D, E, CT scan, Evoked response testing, Cerebrospinal fluid analysis Rationale: There is no definitive diagnostic test for MS. CT scan, evoked response testing, cerebrospinal fluid analysis, and MRI along with history and physical examination are used to establish a diagnosis for MS. EEG and carotid duplex scan are not used for diagnosing MS.

11. In a person having an acute rejection of a transplanted kidney, what would help the nurse understand the course of events (select all that apply)? A. A new transplant should be considered. B. Acute rejection can be treated with OKT3 C. Acute rejection usually leads to chronic rejection. D. Corticosteroids are the most successful drugs used to treat acute rejection. E. Acute rejection is common after a transplant and can be treated with drug therapy.

Answer: B, E Acute rejection can be treated with OKT3, Acute rejection is common after a transplant and can be treated with drug therapy.

Vascular dementia is associated with A. Transient ischemic attacks B. Bacterial or viral infection of neuronal tissue C. Cognitive changes secondary to cerebral ischemia. D. Abrupt changes is cognitive function that are irreversible.

Answer: C Cognitive changes secondary to cerebral ischemia. Rationale: Vascular dementia is the loss of cognitive function that results from ischemic, ischemic-hypoxic, or hemorrhagic brain lesions caused by cardiovascular disease. In this type of dementia, narrowing and blocking of arteries that supply the brain causes a decrease in blood supply.

A patient with rheumatoid arthritis is experiencing articular involvement of the joints. The nurse recognizes that these characteristic changes include (select all that apply) A. Bamboo-shaped fingers. B. Metatarsal head dislocation in feet. C. Noninflammatory pain in large joints. D. Asymmetric involvement of small joints. E. Morning stiffness lasting 60 minutes or more.

Answer: B, E Metatarsal head dislocation in feet, Morning stiffness lasting 60 minutes or more Rationale: Morning stiffness may last from 60 minutes to several hours or more, depending on disease activity. Metatarsal head dislocation and subluxation in the feet may cause pain and walking disability. Joint symptoms occur symmetrically and frequently affect the small joints of the hands (proximal interphalangeal [PIP] and metacarpophalangeal [MCP] joints) and feet (metatarsophalangeal [MTP] joints). Larger peripheral joints such as the wrists, elbows, shoulders, knees, hips, ankles, and jaw may also be involved. Rheumatoid arthritis (RA) is an inflammatory disorder. In early disease, the fingers may become spindle-shaped from synovial hypertrophy and thickening of the joint capsule.

While on a mission trip, the nurse is caring for a patient diagnosed with tetanus. The patient has been given tetanus immune globulin (TIG). What should be the focus of collaborative care (select all that apply)? A. Administration of penicillin B. Tracheostomy for mechanical ventilation C. Administration of polyvalent antitoxin D. Teach correct processing of canned foods. E. Control of spasms with diazepam (Valium)

Answer: B, E Tracheostomy for mechanical ventilation, Control of spasms with diazepam (Valium) Rationale: Control of the spasms of tetanus is essential because the laryngeal and respiratory system spasms cause apnea and anoxia. A tracheostomy is performed early so mechanical ventilation may be done to maintain ventilation. Penicillin is administered for neurosyphilis. Use of polyvalent antitoxin and teaching the correct canning process is done for botulism.

Goals of rehabilitation for the patient with an injury at the C6 level include (select all that apply) A. Stand erect with leg brace B. Feed self with hand devices C. Assist with transfer activities. D. Drive adapted van from wheelchair. E. Push a wheelchair on a flat surface.

Answer: B,C, D, E Rationale: Rehabilitation goals for a patient with a spinal cord injury at the C6 level include the ability to assist with transfer and perform some self-care; feed self with hand devices; push a wheelchair on smooth, flat surfaces; drive an adapted van from a wheelchair; independent computer use with adaptive equipment; and need for attendant care for only 6 hours per day.

Which type of immunity is the result of contact with the antigen through infection and is the longest lasting type of immunity? a. Active innate immunity b. Passive innate immunity c. Active acquired immunity d. Passive acquired immunity

Answer: C Innate immunity is present at birth and its primary role is first-line defense against pathogens. Innate immunity is not antigen specific so it can respond within minutes to an invading microorganism without prior exposure to that organism.

How does interfernon help the body's natural defenses? a. Directly attacks and destroys virus-infected cells b. Augments the immune response by activating phagocytes c. Induces production of antiviral proteins in cells that prevent viral replication d. Is produced by viral infected cells and prevents the transmission of the virus to adjacent cells

Answer: C Interferon produces an antiviral effect in cells by reacting with viruses and inducing the formation of an antiviral protein that prevents new viruses from becoming assembled. Most cytokines are immunomodulatory and do not directly affect antigens, and cytokines such as interleukins may stimulate activation of immune cells.

The nurse explains to the patient with a stroke who is scheduled for angiography that this test is used to determine the A. Presence of increased ICP. B. Site and size of the infarction C. Patency of the cerebral blood vessels D. Presence of blood in the cerebrospinal fluid

Answer: C Rationale: Angiography provides visualization of cerebral blood vessels and can help estimate perfusion and detect filling defects in the cerebral arteries.

A patient with a right-sided hemiplegia and aphasia resulting from a stroke most likely had involvement of the A. Brainstem B. Verterbal artery. C. Left middle cerebral artery D. Right middle cerebral artery

Answer: C Rationale: If the middle cerebral artery is involved in a stroke, the expected clinical manifestations include aphasia, motor and sensory deficit, and hemianopsia on the dominant side and include neglect, motor and sensory deficit, and hemianopsia on the nondominant side.

A patient experience TIAs is scheduled for a carotid endarterectomy. The nurse explains that this procedure is done to A. decrease cerebral edema B. Reduce the brain damage that occurs during a stroke in evolution C. Prevent a stroke by removing atherosclerotic plaques blocking cerebral blood flow. D. Provide a circulatory bypass and thrombotic plaques obstructing cranial circulation.

Answer: C Rationale: In a carotid endarterectomy, the atheromatous lesion is removed from the carotid artery to improve blood flow.

Bladder training in a male patient who has urinary incontinence after a stroke includes A. Limiting fluid intake B. Keeping aurinal in place at all times C. Assisting the patient to stand to void D. Catheterizing the patient every 4 hours.

Answer: C Rationale: In the acute stage of stroke, the primary urinary problem is poor bladder control and incontinence. Nurses should promote normal bladder function and avoid the use of indwelling catheters. A bladder retraining program consists of (1) adequate fluid intake, with most fluids administered between 7:00 AM and 7:00 PM; (2) scheduled toileting every 2 hours with the use of a bedpan, commode, or bathroom; and (3) noting signs of restlessness, which may indicate the need for urination. Intermittent catheterization may be used for urinary retention (not urinary incontinence). During the rehabilitation phase after a stroke, nursing interventions focused on urinary continence include (1) assessment for bladder distention by palpation; (2) offering the bedpan, urinal, commode, or toilet every 2 hours during waking hours and every 3 to 4 hours at night; (3) using a direct command to help the patient focus on the need to urinate; (4) assistance with clothing and mobility; (5) scheduling most fluid intake between 7:00 AM and 7:00 PM; and (6) encouraging the usual position for urinating (i.e., standing for men and sitting for women).

In teaching a patient with Sjogren's syndrome about drug therapy for this disorder, the nurse includes instruction on use of which drug? A. Pregabalin (Lyrica) B. Etanercept (Enbrel) C. Cyclosporine (Restasis) D. Cyclobenzaprine (Flexeril)

Answer: C Cyclosporine Rationale: Cyclosporine (Restasis) ophthalmic drops can be used to treat the chronic dry eye associated with Sjögren's syndrome.

A male patient with a diagnosis of Parkinson's disease (PD) has been admitted recently to a long-term care facility. Which action should the health care team take in order to promote adequate nutrition for this patient? A. Provide multivitamins with each meal. B. Provide a diet that is low in complex carbohydrates and high in protein. C. Provide small, frequent meals throughout the day that are easy to chew and swallow.* D. Provide the patient with a minced or pureed diet that is high in potassium and low in sodium.

Answer: C Rationale: Nutritional support is a priority in the care of individuals with PD. Such patients may benefit from meals that are smaller and more frequent than normal and that are easy to chew and swallow. Multivitamins are not necessary at each meal, and vitamin intake, along with protein intake, must be monitored to prevent contraindications with medications. It is likely premature to introduce a minced or pureed diet, and a low carbohydrate diet is not indicated.

A 65-year-old woman was just diagnosed with Parkinson's disease. The priory nursing intervention is A. Searching for internet for education videos. B. Evaluating the home for environmental safety C. Promoting physical exercise and a well-balanced diet. D. Designing an exercise program to strengthen and stretch specific muscles.

Answer: C Rationale: Promotion of physical exercise and a well-balanced diet are major concerns of nursing care for patients with Parkinson's disease.

The factor related to cerebral blood flow that most often determines the exten of cerebral damage form a stroke is the A. Amount of cardiac output B. Oxygen content of the blood. C. Degree of collateral circulation D. Level of carbon dioxide in the blood.

Answer: C Rationale: The extent of the stroke depends on the rapidity of onset, size of the lesion, and presence of collateral circulation.

A 22-year-old female with paraplegia after a spinal cord injury tells the home care nurse that bowel incontinence occurs two or three times each day. Which action by the nurse is most appropriate? A. Take magnesium citrate (Citroma) every morning with breakfast. B. Teach the patient to gradually increase intake of high-fiber foods. C. Assess bowel movements for frequency, consistency, and volume. D. Instruct the patient to avoid all caffeinated and carbonated beverages.

Answer: C Rationale: The nurse should establish baseline bowel function and explore the patient's current knowledge of an appropriate bowel management program after spinal cord injury. To prevent constipation, caffeine intake should be limited, but not eliminated. Oral saline laxatives such as magnesium citrate are not indicated for long-term management of bowel elimination. Instruction on high-fiber foods is indicated if the patient has a knowledge deficit.

A 25-year-old male patient who is a professional motocross racer has anterior spinal cord syndrome at T10. His history is significant for tobacco, alcohol, and marijuana use. What is the nurse's priority during rehabilitation? A. Prevent urinary tract infections. B. Monitor the patient every 15 minutes. C. Encourage him to verbalize his feelings. D. Teach him about using the gastrocolic reflex.

Answer: C Rationale: To help him with his coping and prevent self-harm, the nurse should create a therapeutic patient environment that encourages his self-expression and verbalization of thoughts and feelings. This patient is at high risk for depression and self-injury because he is likely to lose function below the umbilicus involving lost motor and sensory function. In addition, he is a young adult male patient who is likely to need a wheelchair, have impaired sexual function, and is unlikely to resume his racing career. Because the patient uses tobacco, alcohol, and marijuana frequently, hospitalization is likely to result in a loss of these habits that can make coping especially difficult for him. Prevention of urinary tract infections and facilitating bowel evacuation with the gastrocolic reflex will be important but not as important as helping him cope. In rehabilitation, monitoring every 15 minutes is not needed unless he is on a suicide watch.

The nurse provides dietary instructions to the in-home caregiver of a 45-year-old man who has Huntington's disease. The nurse is most concerned if the caregiver makes which statement? A. "Depression is common and may cause a decrease in appetite." B. "If swallowing becomes difficult, a feeding tube may be needed." C. "Calories should be restricted to prevent unnecessary weight gain." D. "Muscles in the face are affected, and chewing may become impossible."

Answer: C "Calories should be restricted to prevent unnecessary weight gain." Rationale: Patients with Huntington's disease may require 4000 to 5000 calories per day to maintain body weight. Weight loss occurs in patients with Huntington's disease because of choric movements, difficulty swallowing, depression, and mental deterioration.

A female patient's complex symptomatology over the past year has led to a diagnosis of systemic lupus erythematosus (SLE). Which statement demonstrates the patient's need for further teaching about the disease? A. "I'll try my best to stay out of the sun this summer." B. "I know that I probably have a high chance of getting arthritis." C. "I'm hoping that surgery will be an option for me in the future." D. "I understand that I'm going to be vulnerable to getting infections."

Answer: C "I'm hoping that surgery will be an option for me in the future." Rationale: Surgery is not a key treatment modality for SLE, so this indicates a need for further teaching. SLE carries an increased risk of infection, sun damage, and arthritis.

A 62-year-old woman diagnosed with fibromyalgia syndrome (FMS) reports difficulty sleeping at night. Which suggestion should the nurse give to the patient? A. "Drinking a glass of red wine 30 minutes before bedtime will reduce anxiety and help you fall asleep." B. "Evening primrose oil is an herbal supplement that can be used as a sleep aid and to relieve anxiety." C. "Melatonin is a hormone that is often used in supplements to improve sleep and ease fibromyalgia pain." D. "Diphenhydramine (Benadryl) is a nonprescription sleep aid that is effective and does not cause tolerance."

Answer: C "Melatonin is a hormone that is often used in supplements to improve sleep and ease fibromyalgia pain." Rationale: Melatonin is a hormone prepared as a supplement. Scientific evidence suggests that melatonin decreases sleep latency and may increase the duration of sleep. In addition, melatonin may decrease fatigue and pain in individuals with fibromyalgia. Alcohol should not be consumed 4 to 6 hours before bedtime. Evening primrose oil is an herbal product used for breast pain (oral form) and skin disorders (topical form). Long-term use of diphenhydramine for sleep causes tolerance.

A pregnant woman who was tested and diagnosed with HIV infection is very upset. What should the nurse teach this patient about her baby's risk of being born with HIV infection? A. "The baby will probably be infected with HIV." B. "Only an abortion will keep your baby from having HIV." C. "Treatment with antiretroviral therapy will decrease the baby's chance of HIV infection." D. "The duration and frequency of contact with the organism will determine if the baby gets HIV infection."

Answer: C "Treatment with antiretroviral therapy will decrease the baby's chance of HIV infection." Rationale: On average, 25% of infants born to women with untreated HIV will be born with HIV. The risk of transmission is reduced to less than 2% if the infected pregnant woman is treated with antiretroviral therapy. Duration and frequency of contact with the HIV organism is one variable that influences whether transmission of HIV occurs. Volume, virulence, and concentration of the organism as well as host immune status are variables related to transmission via blood, semen, vaginal secretions, or breast milk.

The nurse is caring for a group of patients on a medical unit. After receiving report, which patient should the nurse see first? A. A 42-year-old patient with multiple sclerosis who was admitted with sepsis B. A 72-year-old patient with Parkinson's disease who has aspiration pneumonia C. A 38-year-old patient with myasthenia gravis who declined prescribed medications D. A 45-year-old patient with amyotrophic lateral sclerosis who refuses enteral feedings

Answer: C A 38-year-old patient with myasthenia gravis who declined prescribed medications Rationale:

A female patient complains of a throbbing headache. When her history is obtained, the nurse discovers that the patient has had this type of headache before and experienced photophobia before the headache occurred. The nurse should know that what is probably the cause of this patient's headache? A. Polycythemia vera B. A cluster headache C. A migraine headache D. A hemorrhagic stroke

Answer: C A migraine headache Rationale: Although a headache may occur with any of these options, a migraine headache is the only one that has a throbbing headache with an aura (the photophobia). Headache from polycythemia vera is from erythrocytosis. The cluster headache pain is sharp and stabbing, and the headache with a hemorrhagic stroke has a sudden onset and is not recurrent.

The nurse is providing care for a patient who has been living with HIV for several years. Which assessment finding most clearly indicates an acute exacerbation of the disease? A. A new onset of polycythemia B. Presence of mononucleosis-like symptoms C. A sharp decrease in the patient's CD4+ count D. A sudden increase in the patient's WBC count

Answer: C A sharp decrease in the patient's CD4+ count Rationale: A decrease in CD4+ count signals an exacerbation of the severity of HIV. Polycythemia is not characteristic of the course of HIV. A patient's WBC count is very unlikely to suddenly increase, with decreases being typical. Mononucleosis-like symptoms such as malaise, headache, and fatigue are typical of early HIV infection and seroconversion.

The nurse prepares to administer temozolomide (Temodar) to a 59-year-old white male patient with a glioblastoma multiforme (GBM) brain tumor. What should the nurse assess before giving the medication? A. Serum potassium and serum sodium levels B. Urine osmolality and urine specific gravity C. Absolute neutrophil count and platelet count D. Cerebrosprinal fluid (CSF) pressure and cell count

Answer: C Absolute neutrophil count and platelet count Rationale: Temozolomide causes myelosuppression. The nurse should assess the absolute neutrophil count and the platelet count. The absolute neutrophil count should be >1500/μL and platelet count >100,000/μL.

Which nursing diagnosis is likely to be a priority in the care of a patient with myasthenia gravis (MG)? A. Acute confusion B. Bowel incontinence C. Activity intolerance D. Disturbed sleep pattern

Answer: C Activity intolerance Rationale: The primary feature of MG is fluctuating weakness of skeletal muscle. Bowel incontinence and confusion are unlikely signs of MG, and although sleep disturbance is likely, activity intolerance is usually of primary concern.

The patient developed gout while hospitalized for a heart attack. When doing discharge teaching for this patient who takes aspirin for its antiplatelet effect, what should the nurse include about preventing future attacks of gout? A. Limit fluid intake. B. Administration of probenecid (Benemid) C. Administration of allopurinol (Zyloprim) D. Administration of nonsteroidal antiinflammatory drugs (NSAIDs)

Answer: C Administration of allopurinol (Zyloprim) Rationale: To prevent future attacks of gout, the urate-lowering drug allopurinol may be administered. Increased fluid will be encouraged to prevent precipitation of uric acid in the renal tubules. This patient will not be able to take the uricosuric drug probenecid because the aspirin the patient must take will inactivate its effect, resulting in urate retention. NSAIDs for pain management will not be used, related to the aspirin, because of the potential for increased side effects.

A patient's sudden onset of hemiplegia has necessitated a computed tomography (CT) of her head. Which assessment should the nurse complete prior to this diagnostic study? A. Assess the patient's immunization history. B. Screen the patient for any metal parts or a pacemaker. C. Assess the patient for allergies to shellfish, iodine, or dyes. D. Assess the patient's need for tranquilizers or antiseizure medications.

Answer: C Assess the patient for allergies to shellfish, iodine, or dyes. Rationale: Allergies to shellfish, iodine, or dyes contraindicate the use of contrast media in CT. The patient's immunization history is not a central consideration, and the presence of metal in the body does not preclude the use of CT as a diagnostic tool. The need to assess for allergies supersedes the need for tranquilizers or antiseizure medications in the majority of patients.

The physician orders intracranial pressure (ICP) readings every hour for a 23-year-old male patient with a traumatic brain injury from a motor vehicle crash. The patient's ICP reading is 21 mm Hg. It is most important for the nurse to take which action? A. Document the ICP reading in the chart. B. Determine if the patient has a headache. C. Assess the patient's level of consciousness. D. Position the patient with head elevated 60 degrees

Answer: C Assess the patient's level of consciousness Rationale: The patient has an increased ICP (normal ICP ranges from 5 to 15 mm Hg). The most sensitive and reliable indicator of neurologic status is level of consciousness. The Glasgow Coma Scale may be used to determine the degree of impaired consciousness.

The nurse is alerted to a possible acute subdrual hematoma in the patient who A. Has a linear skull fracture crossing a major artery. B. Has focal symptoms of brain damage with no recollection of a head injury C. Develops decreased level of consciousness and headache within 48 hours of a head injury D. Has an immediate loss of consciousness with a brief lucid interval followed by decreasing level of consciousness

Answer: C Develops decreased level of consciousness and headache within 48 hours of a head injury Rationale: An acute subdural hematoma manifests within 24 to 48 hours of the injury. The signs and symptoms are similar to those associated with brain tissue compression in elevated ICP and include decreasing level of consciousness and headache.

The nurse is caring for a patient who has osteoarthritis (OA) of the knees. The nurse teaches the patient that the most beneficial measure to protect the joints is to do what? A. Use a wheelchair to avoid walking as much as possible. B. Sit in chairs that cause the hips to be lower than the knees. C. Eat a well-balanced diet to maintain a healthy body weight. D. Use a walker for ambulation to relieve the pressure on the hips.

Answer: C Eat a well-balanced diet to maintain a healthy body weight. Rationale: Because maintaining an appropriate load on the joints is essential to the preservation of articular cartilage integrity, the patient should maintain an optimal overall body weight or lose weight if overweight. Walking is encouraged. The chairs that would be best for this patient have a higher seat and armrests to facilitate sitting and rising from the chair. Relieving pressure on the hips is not important for OA of the knees.

A 42-year-old man is admitted to the hospital with a diagnosis of Guillain-Barré syndrome. The physician orders include IV Sandoglobulin. What is important for the nurse to assess for before administration? A. Elevated fasting blood glucose and serum albumin B. Elevated activated partial thromboplastin time (aPTT) C. Elevated serum creatinine and blood urea nitrogen (BUN) D. Elevated aspartate aminotransferase and alanine aminotransferase

Answer: C Elevated serum creatinine and blood urea nitrogen (BUN) Rationale: Patients receiving IV administration of high-dose immunoglobulin (Sandoglobulin) need to be well hydrated and have adequate renal function. Elevated serum creatinine and blood urea nitrogen indicate impaired renal function.

8. A patient is undergoing plasmapheresis for treatment of systemic lupus erythematosus. The nurse explains that plasmapheresis is used in her treatment to A. Remove T lymphocytes in her blood that are producing antinuclear antibodies. B. Remove normal particles in her blood that are being damaged by autoantibodies. C. Exchange her plasma that contains antinuclear antibodies with a substitute fluid. D. Replace viral-damaged cellular components of her blood with replacement whole blood.

Answer: C Exchange her plasma that contains antinuclear antibodies with a substitute fluid. Rationale: The rationale for performing therapeutic plasmapheresis in patients with autoimmune disorders such as SLE is to remove pathologic substances (i.e., antinuclear antibodies) from plasma.

A patient arrives to the emergency department with a reported overdose of diazepam (Valium). Which medication would the nurse prepare to administer as an antidote? A. Protamine sulfate B. Naloxone (Narcan) C. Flumazenil (Romazicon) D. Phytonadione (AquaMephyton)

Answer: C Flumazenil (Romazicon) Rationale: Flumazenil reverses the effects of benzodiazepines, such as diazepam (Valium), by acting as a competitive antagonist at gamma-aminobutyric acid (GABA)-benzodiazepine receptor sites. Protamine sulfate reverses the anticoagulant effects of heparin. Naloxone (Narcan) is an opioid antagonist and used with opioid overdose. Phytonadione (AquaMephyton) is Vitamin K and used for hypoprothrombinemia caused by anticoagulants such as warfarin (Coumadin).

Which nursing intervention is most appropriate when caring for patients with dementia? A. Avoid direct eye contact. B. Lovingly call the patient "honey" or "sweetie." C. Give simple directions, focusing on one thing at a time. D. Treat the patient according to his or her age-related behavior.

Answer: C Give simple directions, focusing on one thing at a time. Rationale: When dealing with patients with dementia, tasks should be simplified, giving directions using gestures or pictures and focusing on one thing at a time. It is best to treat these patients as adults, with respect and dignity, even when their behavior is childlike. The nurse should use gentle touch and direct eye contact. Calling the patient "honey" or "sweetie" can be condescending and does not demonstrate respect.

The new patient has a diagnosis of frontal lobe dementia. What functional difficulties should the nurse expect in this patient? A. The lack of reflexes B. Endocrine problems C. Higher cognitive function abnormalities D. Respiratory, vasomotor, and cardiac dysfunction

Answer: C Higher cognitive function abnormalities Rationale: Because the frontal lobe is responsible for higher cognitive function, this patient may have difficulty with memory retention, voluntary eye movements, voluntary motor movement, and expressive speech. The lack of reflexes would occur if the patient had problems with the reflex arcs in the spinal cord. Endocrine problems would be evident if the hypothalamus or pituitary gland were affected. Respiratory, vasomotor, and cardiac dysfunction would occur if there were a problem in the medulla.

A patient has a systemic blood pressure of 120/60 and an ICP of 24 mm Hg. After calculating the patient's cerebral perfusion pressure (CPP), how does the nurse interpret the results? A. High blood flow to the brain B. Normal intracranial pressure C. Impaired blood flow to the brain D. Adequate autoregulation of blood flow

Answer: C Impaired blood flow to the brain Rationale: Normal CPP is 60 to 100 mm Hg. The CPP is calculated with mean arterial pressure (MAP) minus ICP. MAP = SBP + 2 (DBP)/ 3: 120 mm Hg + 2 (60 mm Hg)/3 = 80 mm Hg. MAP - ICP: 80mm Hg - 24 mm Hg = 56 mm Hg CPP. The decreased CPP indicates that there is impaired cerebral blood flow and that autoregulation is impaired. Because the ICP is 24, it is elevated and requires treatment.

The nurse is monitoring the effectiveness of antiretroviral therapy (ART) for a 56-year-old man with acquired immunodeficiency syndrome (AIDS). What laboratory study result indicates the medications have been effective? A. Increased viral load B. Decreased neutrophil count C. Increased CD4+ T cell count D. Decreased white blood cell count

Answer: C Increased CD4+ T cell count Rationale: Antiretroviral therapy is effective if there are decreased viral loads and increased CD4+ T cell counts.

The patient with an autoimmune disease will be treated with plasmapheresis. What should the nurse teach the patient about this treatment? A. It will gather platelets for use later when needed. B. It will cause anemia because it removes whole blood and RBCs are damaged. C. It will remove the IgG autoantibodies and antigen complexes from the plasma. D. It will remove the peripheral stem cells in order to cure the autoimmune disease.

Answer: C It will remove the IgG autoantibodies and antigen complexes from the plasma. Rationale: Plasmapheresis removes plasma that contains autoantibodies (usually IgG class) and antigen-antibody complexes to remove the pathologic substances in the plasma without causing anemia. Plateletpheresis removes platelets from normal individuals for use by patients with low platelet counts. Apheresis is used to collect stem cells from peripheral blood that does not cure autoimmune disease.

A 21-year-old student had taken amoxicillin once as a child for an ear infection. She is given an injection of Penicillin V and develops a systemic anaphylactic reaction. What manifestations would be seen first? A. Dyspnea B. Dilated pupils C. Itching and edema D. Wheal-and-flare reaction

Answer: C Itching and edema Rationale: A systemic anaphylactic reaction starts with edema and itching at the site of exposure to the antigen. Shock can rapidly develop with rapid, weak pulse; hypotension; dilated pupils; dyspnea, and possible cyanosis. The wheal-and-flare reaction occurs with a localized anaphylactic reaction such as a mosquito bite.

An obstruction of the anterior cerebral arteries with affect functions of A. Visual imaging B. Balance and coordination C. Judgement, insight, and reasoning D. Visual and auditory integration for language comprehension.

Answer: C Judgement, insight, and reasoning Rationale: The anterior cerebral artery feeds the medial and anterior portions of the frontal lobes. The anterior portion of the frontal lobe controls higher order processes such as judgment and reasoning.

Screening for HIV infection generally involves A. Laboratory analysis of blood to detect HIV antigen. B. Electrophoretic analysis for HIV antigen in plasma. C. Laboratory analysis of blood to detect HIV antibodies. D. Analysis of lymph tissues for the presence of HIV RNA.

Answer: C Laboratory analysis of blood to detect HIV antibodies. Rationale: The most useful screening tests for HIV detect HIV-specific antibodies.

Unlicensed assistive personnel (UAP) working for a home care agency report a change in the alertness and language of an 82-year-old female patient. The home care nurse plans a visit to evaluate the patient's cognitive function. Which assessment would be most appropriate? A. Glasgow Coma Scale (GCS) B. Confusion Assessment Method (CAM) C. Mini-Mental State Examination (MMSE) D. National Institutes of Health Stroke Scale (NIHSS)

Answer: C Mini-Mental State Examination (MMSE) Rationale: The MMSE is a commonly used tool to assess cognitive function. Cognitive testing is focused on evaluating memory, ability to calculate, language, visual-spatial skills, and degree of alertness. The CAM is used to assess for delirium. The GCS is used to assess the degree of impaired consciousness. The NIHSS is a neurologic examination stroke scale used to evaluate the effect of acute cerebral infarction on the levels of consciousness, language, neglect, visual field loss, extraocular movement, motor strength, ataxia, dysarthria, and sensory loss.

A patient has begun immunotherapy for the treatment of intractable environmental allergies. When administering the patient's immunotherapy, what is the nurse's priority action? A. Monitor the patient's fluid balance. B. Assess the patient's need for analgesia. D. Monitor for signs and symptoms of an adverse reaction. C. Assess the patient for changes in level of consciousness.

Answer: C Monitor for signs and symptoms of an adverse reaction. Rationale: When administering immunotherapy, it is imperative to closely monitor the patient for any signs of an adverse reaction. The high risk and significant consequence of an adverse reaction supersede the need to assess the patient's fluid balance. Pain and changes in level of consciousness are not likely events when administering immunotherapy.

Opportunistic diseases in HIV infection A. Are usually benign B. Are generally slow to develop and progress C. Occur in the presence of immunosuppression. D. Are curable with appropriate drug interventions.

Answer: C Occur in the presence of immunosuppression. Rationale: Management of HIV infection is complicated by the many opportunistic diseases that can develop as the immune system deteriorates (see Table 15-10).

The nurse is caring for a group of well older people at a community day center. Which neurologic finding associated with aging would the nurse expect to find in older adults? A. Longer reaction time B. Improved sense of taste C. Orthostatic hypotension D. Hyperactive deep tendon reflexes

Answer: C Orthostatic hypotension Rationale: Older adults are more likely to experience orthostatic hypotension related to altered coordination of neuromuscular activity. Other neurologic changes in the older adult include atrophy of taste buds with decreased sense of taste, below average reflex score (and diminished deep tendon reflexes), and slowed reaction times.

The nurse on the clinical unit is assigned to four patients. Which patient should she assess first? A. Patient with a skull fracture who nose is bleeding B. Older patient with a stroke who is confused and whose daughter is present C. Patient with meningitis who is suddenly agitated reporting a headache of 10 on a 0-to-10 scale D. Patient who had a craniotomy for a rain tumor who is now 3 days postoperative and has had continued vomiting.

Answer: C Patient with meningits who suddently agitated reporting a headache of 10 on a 0-to-10 scale. Rationale: The patient with meningitis should be seen first; patients with meningitis must be observed closely for manifestations of elevated ICP, which is thought to result from swelling around the dura and increased cerebrospinal fluid (CSF) volume. Sudden change in the level of consciousness or change in behavior along with a sudden severe headache may indicate an acute elevation of ICP. The patient who has undergone cranial surgery should be seen second; although nausea and vomiting are common after cranial surgery, it can result in elevations of ICP. Nausea and vomiting should be treated with antiemetics. The patient with a skull fracture needs to be evaluated for CSF leakage occurring with the nose bleed and should be seen third. Confusion after a stroke may be expected; the patient should have a family member present.

A nurse is assessing the recent health history of a 63-year-old patient with osteoarthritis (OA). The nurse determines that the patient is trying to manage the condition appropriately when the patient describes which activity pattern? A. Bed rest with bathroom privileges B. Daily high-impact aerobic exercise C. Regular exercise program of walking D. Frequent rest periods with minimal exercise

Answer: C Regular exercise program of walking Rationale: A regular low-impact exercise, such as walking, is important in helping to maintain joint mobility in the patient with osteoarthritis. A balance of rest and activity is needed. High-impact aerobic exercises would cause stress to affected joints and further damage.

The nurse is planning psychosocial support for the patient and family of the patient who suffered a stroke. What factor will most likely have the greatest impact on positive family coping with the situation? A. Specific patient neurologic deficits B. The patient's ability to communicate C. Rehabilitation potential of the patient D. Presence of complications of a stroke

Answer: C Rehabilitation potential of the patient Rationale: Although a patient's neurologic deficit might initially be severe after a stroke, the ability of the patient to recover is most likely to positively impact the family's coping with the situation. Providing explanations and emotional support beginning in the acute phase through the rehabilitation phase will facilitate coping. Emphasizing successes will offer the most realistic hope for the patient's rehabilitation and helps maintain hope for the patient's future abilities.

The nurse is providing care for a patient who has been diagnosed with Guillain-Barré syndrome. Which assessment should be the nurse's priority? A. Pain assessment B. Glasgow Coma Scale C. Respiratory assessment D. Musculoskeletal assessment

Answer: C Respiratory Assessment Rationale: Although all of the assessments are necessary in the care of patients with Guillain-Barré syndrome, the acute risk of respiratory failure necessitates vigilant monitoring of the patient's respiratory status.

Which sensory-perceptual deficit is associated with left-sided stroke (right hemiplegia)? A. Overestimation of physical abilities B. Difficulty judging position and distance C. Slow and possibly fearful performance of tasks D. Impulsivity and impatience at performing tasks

Answer: C Slow and possibly fearful performance of tasks Rationale: Patients with a left-sided stroke (right hemiplegia) commonly are slower in organization and performance of tasks and may have a fearful, anxious response to a stroke. Overconfidence, spatial disorientation, and impulsivity are more commonly associated with a right-sided stroke.

2. One function of cell-mediated immunity is A. formation of antibodies. B. Activation of the complement system. C. Surveillance for malignant cell changes. D. Opsonizaiton of antigens to allow phagocytosis by neutrophils.

Answer: C Surveillance for malignant cell changes. Rationale: One role of cell-mediated immunity is immune surveillance to detect any malignant changes in cells and then destroy them.

A patient with a C7 spinal cord injury undergoing rehabilitation tells the nurse he must have the flu because he has a bad headache and nausea. The nurse's first priority is to A. Call the physician. B. Check the patient's temperature. C. Take the patient's blood pressure. D. Elevate the head of the bed to 90 degrees

Answer: C Take the patient's blood pressure. Rationale: Autonomic dysreflexia is a massive, uncompensated cardiovascular reaction mediated by the sympathetic nervous system. Manifestations include hypertension (up to 300 mm Hg systolic), throbbing headache, marked diaphoresis above the level of the injury, bradycardia (30 to 40 beats/min), piloerection, flushing of the skin above the level of the injury, blurred vision or spots in the visual fields, nasal congestion, anxiety, and nausea. It is important to measure blood pressure when a patient with a spinal cord injury complains of headache. Other nursing interventions in this serious emergency are elevation of the head of the bed 45 degrees or sitting the patient upright, notification of the physician, and assessment to determine the cause. Table 61-8 lists the causes and symptoms of autonomic dysreflexia. The nurse must monitor blood pressure frequently during the episode. An α-adrenergic blocker or an arteriolar vasodilator may be administered.

8. During neurologic testing, the patient is able to perceive pain elicited by pinprick. Based on this finding, the nurse may omit testing for A. Position sense B. Patellar reflexes. C. Temperature perception D. Heel-to-shin movements

Answer: C Temperature perception Rationale: If pain sensation is intact, assessment of temperature sensation may be omitted because both sensations are transmitted by the same ascending pathways.

During HIV infection A. The virus replicates mainly in B-cells before spreading to CD4 T cells. B. infection of monocytes may occur, but antibodies quickly destroy these cells. C. The immune system is impaired predominantly by the eventual widespread destruction of CD4 T cells. D. A long period of dormancy develops during which HIV cannot be found in the blood and there is little viral replication.

Answer: C The immune system is impaired predominantly by the eventual widespread destruction of CD4 T cells. Rationale: Immune dysfunction in HIV disease is caused predominantly by damage to and destruction of CD4+ T cells (i.e., T helper cells or CD4+ T lymphocytes).

The nurse is preparing the patient for an electromyography (EMG). What should the nurse include in teaching the patient before the test? A. The patient will be tilted on a table during the test. B. It is noninvasive, and there is no risk of electric shock. C. The pain that occurs is from the insertion of the needles. D. The passive sensor does not make contact with the patient.

Answer: C The pain that occurs is from the insertion of the needles Rationale: With an EMG, pain may occur when needles are inserted to record the electrical activity of nerve and skeletal muscle. The patient is tilted on a table during a myelogram. The electroencephalography (EEG) is noninvasive without a danger of electric shock. The magnetoencephalography (MEG) is done with a passive sensor that does not make contact with the patient.

Which T lymphocytes are involved in direct attack and destruction of foreign pathogens? a. Dendritic cells b. Natural killer cells c. T helper (CD4) cells d. T cytotoxic (CD8) cells

Answer: D T Cytotoxic cells directly attack antigens on the cell membrane of foreign pathogens and release cytolytic substances that destroy pathogens. Dendritic cells primarily capture antigens at sites of contact with the external environment and then transport the antigen to a T cell with specificity for the antigen. Natural killer cells are involved in cell-mediated immunity but are not considered T lymphocytes. CD4 cells (T helper cells) are involved in the regulation of cell-mediated immunity and humoral antibody response.

In planning long-term care for a patient after a craniotomy, what must the nurse include when teaching the patient, family, and caregiver? A. Seizure disorders may occur in weeks or months. B. The family will be unable to cope with role reversals. C. There are often residual changes in personality and cognition. D. Referrals will be made to eliminate residual deficits from the damage.

Answer: C There are often residual changes in personality and cognition Rationale: In long-term care planning, the nurse must include the family and caregiver when teaching about potential residual changes in personality, emotions, and cognition as these changes are most difficult for the patient and family to accept. Seizures may or may not develop. The family and patient may or may not be able to cope with role reversals. Although residual deficits will not be eliminated with referrals, they may be improved.

The patient with increased ICP from a brain tumor is being monitored with a ventriculostomy. What nursing intervention is the priority in caring for this patient? A. Administer IV mannitol (Osmitrol). B. Ventilator use to hyperoxygenate the patient C. Use strict aseptic technique with dressing changes. D. Be aware of changes in ICP related to leaking CSF.

Answer: C Use strict aseptic technique with dressing changes Rationale: The priority nursing intervention is to use strict aseptic technique with dressing changes and any handling of the insertion site to prevent the serious complication of infection. IV mannitol (Osmitrol) or hypertonic saline will be administered as ordered. Ventilators may be used to maintain oxygenation. CSF leaks may cause inaccurate ICP readings, or CSF may be drained to decrease ICP, but strict aseptic technique to prevent infection is the nurse's priority of care.

The patient with diabetes mellitus has been ill for some time with a severe lung infection needing corticosteroids and antibiotics. The patient does not feel like eating. The nurse understands that this patient is likely to develop A. major histoincompatibility. B. primary immunodeficiency. C. secondary immunodeficiency. D. acute hypersensitivity reaction.

Answer: C secondary immunodeficiency Rationale: Secondary immunodeficiency is most commonly caused by immunosuppressive drugs, such as corticosteroids. It can also be caused by diabetes mellitus, severe infection, malnutrition, and chronic stress, all of which are present in this patient. The other options are not possible for this patient. Histoincompatibility occurs when the human leukocyte antigen (HLA) system of the donor is not compatible with the recipient's HLA genes. Primary immunodeficiency is rare and includes phagocytic defects, B cell deficiency, T cell deficiency, or a combination of B cell and T cell deficiency. Acute hypersensitivity reaction is an anaphylactic-type allergic reaction to an antigen.

A 24-year-old woman is admitted to the emergency department for management of a cocaine overdose. What assessment findings will the nurse anticipate? A. Fatigue, sleepiness, and depression B. Lethargy, pinpoint pupils, and nystagmus C. Palpitations, tachycardia, and hypertension D. Cold skin, decreased oxygen saturation, and bradypnea

Answer: C Palpitations, tachycardia, and hypertension Rationale: Cardiovascular assessment findings indicating cocaine toxicity include palpitations, tachycardia, hypertension, and dysrhythmias. Central nervous system assessment findings include a feeling of impending doom, euphoria, agitation, combativeness, seizures, hallucinations, confusion, paranoia, and fever.

A 19-year-old male being tested for multiple allergies develops localized redness and swelling in reaction to a patch skin test. Which intervention by the nurse would have the highest priority? A. Notify the primary care provider B. Apply a topical anti-inflammatory cream C. Remove the patch and extract from the skin D. Administer oral diphenhydramine (Benadryl)

Answer: C Remove the patch and extract from the skin. Rationale: If a severe reaction to a patch skin test occurs, the nurse should immediately remove the patch and the extract from the skin. Next the nurse should apply a topical anti-inflammatory cream to the site. A subcutaneous injection of epinephrine may also be necessary but would need a health care provider's order.

The nurse is caring for a patient admitted to the hospital with pneumonia who has a history of misuse of alprazolam (Xanax). Which clinical manifestations could be attributed to withdrawal from this medication (select all that apply)? A. Anxiety B. Tremors C. Seizures D. Delirium E. Drowsiness

Answer: C, D Seizures, Delirium Rationale: Alprazolam (Xanax) is a sedative-hypnotic. Withdrawal symptoms of a sedative-hypnotic include weakness, restlessness, insomnia, hyperthermia, orthostatic hypotension, confusion, disorientation, seizures, psychotic delirium, exhaustion, possible respiratory and cardiac arrest, and death. Tremors occur in withdrawal from opioids. Anxiety is seen in withdrawal from cannabis. Drowsiness would be an effect of taking alprazolam, not withdrawing from it.

The patient with peripheral facial paresis on the left side of her face is diagnosed with Bell's palsy. What should the nurse include in teaching the patient about self-care (select all that apply)? A. Administration of antiseizure medications B. Preparing for a nerve block to relieve pain C. Administration of corticosteroid medications D. Dark glasses and artificial tears to protect the eyes E. Surgeries available if conservative therapy is not effective

Answer: C, D Administration of corticosteroid medications & Dark glasses and artificial tears to protect the eyes Rationale: Self-care for Bell's palsy includes corticosteroid medications to decrease inflammation of the facial nerve (CNVII) and protecting the cornea from drying out because of the inability to close the eyelid. Antiseizure medications, a nerve block, or surgeries are used for trigeminal neuralgia.

The woman is afraid she may get HIV from her bisexual husband. What should the nurse include when teaching her about preexposure prophylaxis (select all that apply)? A. Take fluconazole (Diflucan). B. Take amphotericin B (Fungizone). C. Use condoms for risk-reducing sexual relations. D. Take emtricitabine and tenofovir (Truvada) regularly. E. Have regular HIV testing for herself and her husband.

Answer: C, D, E Use condoms for risk-reducing sexual relations, Take emtricitabine and tenofovir (Truvada) regularly, & Have regular HIV testing for herself and her husband. Rationale: Using male or female condoms, having monthly HIV testing for the patient and her husband, and the woman taking emtricitabine and tenofovir regularly has shown to decrease the infection of heterosexual women having sex with a partner who participates in high-risk behavior. Fluconazole and amphotericin B are taken for Candida albicans, Coccidioides immitis, and Cryptococcosus neoformans, which are all opportunistic diseases associate with HIV infection.

The patient tells the nurse that he has used cannabis regularly for the last 20 years. What should the nurse expect to find in the history and physical assessment (select all that apply)? A. Gastric ulcers B. Violent behavior C. Productive cough D. Memory impairment E. Crackles in the lungs F. Coronary artery disease

Answer: C, D, E Productive cough, Memory impairment, Crackles in the lungs Rationale: Smoking cannabis presents similar health risks to smoking tobacco. The risk for bronchitis increases with the deeper inhalation and longer holding of the smoke when smoking cannabis compared to cigarette smoking. The ability to learn and remember is impaired by regular cannabis use. Gastric ulcers are more likely to be seen with regular use of opioids. Violent behavior is more likely with amphetamines. Coronary artery disease would be increased with the regular use of nicotine.

Nursing management of a patient with a brain tumor includes (select all that apply) A. Discussing with the patient methods to control inappropriate behavior B. Using diversion techniques to keep the patient stimulated and motivated C. Assisting and supporting the family in understanding any changes in behavior D. Limiting self-care activities until the patient has regained maximum physical functioning E. Planning for seizure precautions and teaching the patient and the caregiver about anti-seizure drugs.

Answer: C, E, Assisting and supporting the family in understanding any changes in behavior & Planning for seizure precautions and teaching the patient and the caregiver about anti-seizure drugs. Rationale: Nursing interventions should be based on a realistic appraisal of the patient's condition and prognosis after cranial surgery. The nurse should provide support and education to the caregiver and family about the patient's behavioral changes. The nurse should be prepared to manage seizures and teach the caregiver and family about antiseizure medications and how to manage a seizure. An overall goal is to foster the patient's independence for as long as possible and to the highest degree possible. The nurse should decrease stimuli in the patient's environment to prevent increases in intracranial pressure.

Which immunoglobulin is responsible for the primary immune response and forms antibodies to ABO blood antigens? a. IgA b. IgD c. IgG d. IgM

Answer: D IgM immunoglobulin is predominant in the primary immune response and produces antibodies against ABO blood antigens.

3. Information provided by the patient that would help differentiate a hemorrhagic stroke form a thrombotic stroke includes A. Sensory distrubance B. A history of hypertension C. Presence of motor weakness D. Suddent onset of severe headache

Answer: D Rationale: A hemorrhagic stroke usually causes a sudden onset of symptoms, which include neurologic deficits, headache, nausea, vomiting, decreased level of consciousness, and hypertension. Ischemic stroke symptoms may progress in the first 72 hours as infarction and cerebral edema increase.

Of the following patients, the nurse recognizes that the one with the highest risk for a stroke is a(n) A. Obese 45-year-old Native American B. 35-year-old Asian American woman who smokes C. 32-year old white woman taking oral contraceptives D. 65-year old African American with hypertension.

Answer: D Rationale: Nonmodifiable risk factors for stroke include age (older than 65 years), male gender, ethnicity or race (incidence is highest in African Americans; next highest in Hispanics, Native Americans/Alaska Natives, and Asian Americans; and next highest in white people), and family history of stroke or personal history of a transient ischemic attack or stroke. Modifiable risk factors for stroke include hypertension (most important), heart disease (especially atrial fibrillation), smoking, excessive alcohol consumption (causes hypertension), abdominal obesity, sleep apnea, metabolic syndrome, lack of physical exercise, poor diet (high in saturated fat and low in fruits and vegetables), and drug abuse (especially cocaine). Other risk factors for stroke include a diagnosis of diabetes mellitus, increased serum levels of cholesterol, birth control pills (high levels of progestin and estrogen), history of migraine headaches, inflammatory conditions, hyperhomocystinemia, and sickle cell disease.

Which patient is most at risk for developing delirium? A. A 50_year-old women with cholecystitis B. A 19-year old man with a fractured femur C. A 42-year-old woman having an elective hysterectomy D. A 78-year-old man admitted to the medical unit with complications related to heart failure.

Answer: D Rationale: Risk factors that can precipitate delirium include age of 65 years or older, male gender, and severe acute illness (e.g., heart failure). The 78-year-old man has the most risk factors for delirium (see Table 60-14).

The nurse is reinforcing teaching with a newly diagnosed patient with amyotrophic lateral sclerosis. Which statement would be appropriate to include in the teaching? A. "ALS results from an excess chemical in the brain, and the symptoms can be controlled with medication." B. "Even though the symptoms you are experiencing are severe, most people will recover with treatment." C. "You need to consider advance directives now, since you will lose the cognitive function as the disease progresses." D "This is a progressing disease that eventually results in permanent paralysis, though you will not lose any cognitive function."

Answer: D Rationale: The disease results in destruction of the motor neurons in the brainstem and spinal cord, causing gradual paralysis. Cognitive function is maintained. Because there is no cure for amyotrophic lateral sclerosis (ALS), collaborative care is palliative and based on symptom relief. Death usually occurs within 3-6 years after diagnosis.

The nurse finds that an 87-year-old woman with Alzhimer's disease is continually rubbing, flexing, and kicking out her legs throughout the day. The night shift reports that this same behavior escalates at night, preventing her form obtaining her required sleep. The next step the should should take is to A. Ask the physician for a daytime sedative for the patient. B. Request soft restraints to prevent her from falling out of her bed. C. Ask the physician for a night time sleep medication for the patient. D. assess the patient more closely, suspecting a disorder such as restless leg syndrome.

Answer: D Rationale: The severity of sensory symptoms of restless legs syndrome (RLS) ranges from infrequent, minor discomfort (paresthesias, including numbness, tingling, and "pins and needles" sensation) to severe pain. The discomfort occurs when the patient is sedentary and is most common in the evening or at night. The pain at night can disrupt sleep and is often relieved by physical activity, such as walking, stretching, rocking, or kicking. In the most severe cases, patients sleep only a few hours at night, which results in daytime fatigue and disruption of the daily routine. The motor abnormalities associated with RLS consist of voluntary restlessness and stereotyped, periodic, involuntary movements. The involuntary movements usually occur during sleep. Symptoms are aggravated by fatigue.

The nurse is teaching a senior citizen's group about signs and symptoms of a stroke. Which statement by the nurse would provide accurate information? A. "Take the person to the hospital if a headache lasts for more than 24 hours." B. "Stroke symptoms usually start when the person is awake and physically active." C. "A person with a transient ischemic attack has mild symptoms that will go away." D. "Call 911 immediately if a person develops slurred speech or difficulty speaking."

Answer: D "Call 911 immediately if a person develops slurred speech or difficulty speaking." Rationale: Medical assistance should be obtained immediately for someone with signs and symptoms of a stroke such as sudden numbness; weakness; paralysis of the face, arm, or leg (especially on one side of the body); sudden confusion; trouble speaking or understanding; slurred speech; sudden trouble seeing in one or both eyes; sudden trouble walking; dizziness; loss of balance or coordination; or a sudden, severe headache with no known cause. A person with signs and symptoms of a transient ischemic attack should seek medical attention immediately because it is unknown if the symptoms will resolve or persist and progress to a stroke. Onset of signs and symptoms of a stroke vary depending on the type. Onset of an ischemic thrombotic stroke usually occurs at rest. Onset of an ischemic embolic stroke is not related to rest or activity, and onset of a hemorrhagic stroke usually occurs with activity.

The nurse obtains a history from a 46-year-old woman with rheumatoid arthritis. It is most important for the nurse to follow up on which patient statement? A. "I perform range of motion exercises at least twice a day." B. "I use a heating pad for 20 minutes to reduce morning stiffness." C. "I take a 20-minute nap in the afternoon even if I sleep 9 hours at night." D. "I restrict fluids to prevent edema when taking methotrexate (Rheumatrex)."

Answer: D "I restrict fluids to prevent edema when taking methotrexate (Rheumatrex)." Rationale: Methotrexate can affect renal function. Patients should be well hydrated to prevent nephropathy. Heat application, range of motion, and rest are appropriate interventions to manage rheumatoid arthritis.

The nurse is teaching a 24-year-old female patient who has a latex allergy about preventing and treating allergic reactions. Which statement, if made by the patient, indicates a need for further teaching? A. "My dentist should be told about my latex allergy." B. "I should avoid foods such as bananas, avocados, and kiwi." C. "I will use vinyl gloves for activities such as housekeeping." D. "Because my reactions are not severe, I will not need an EpiPen."

Answer: D "Because my reactions are not severe, I will not need an EpiPen." Rationale: An individual with latex allergies should carry an injectable epinephrine pen. The proteins in latex are similar to the proteins in certain foods and may cause an allergic reaction in people who are allergic to latex. Foods to avoid include banana, avocado, chestnut, kiwi, tomato, water chestnuts, guava, hazelnuts, potatoes, peaches, grapes, and apricots. Vinyl gloves are not latex and are safe to use. Individuals with latex allergies need to share this information with all health care providers and wear a medical alert bracelet.

Which patient statement most clearly suggests a need to assess the patient for ankylosing spondylitis (AS)? A. "My right elbow has become red and swollen over the last few days." B. "I wake up stiff every morning, and my knees just don't want to bend." C. "My husband tells me that my posture has become so stooped this winter." D. "My lower back pain seems to be getting worse all the time, and nothing seems to help."

Answer: D "My lower back pain seems to be getting worse all the time, and nothing seems to help." Rationale: AS primarily affects the axial skeleton. Based on this, symptoms of inflammatory spine pain are often the first clues to a diagnosis of AS. Knee or elbow involvement is not consistent with the typical course of AS. Back pain is likely to precede the development of kyphosis.

Which statement by the patient who has had an organ transplant would indicate that the patient understands the teaching about the immunosuppressive medications? A. "My drug dosages will be lower because the medications enhance each other." B. "Taking more than one medication will put me at risk for developing allergies." C. "I will be more prone to malignancies because I will be taking more than one drug." D. "The lower doses of my medications can prevent rejection and minimize the side effects."

Answer: D "The lower doses of my medications can prevent rejection and minimize the side effects." Rationale: Because immunosuppressants work at different phases of the immune response, lower doses of each drug can be used to produce effective immunosuppression while minimizing side effects. The use of several medications is not because they enhance each other and does not increase the risk of allergies or of malignancies.

After learning about rehabilitation for his spinal cord tumor, which statement shows the patient understands what rehabilitation is and can do for him? A. "I want to be rehabilitated for my daughter's wedding in 2 weeks." B. "Rehabilitation will be more work done by me alone to try to get better." C. "I will be able to do all my normal activities after I go through rehabilitation." D "With rehabilitation, I will be able to function at my highest level of wellness."

Answer: D "With rehabilitation, I will be able to function at my highest level of wellness." Rationale: Rehabilitation is an interdisciplinary endeavor carried out with a team approach to teach and enable the patient to function at the patient's highest level of wellness and adjustment. It will be a lot of work for all involved and take longer than 2 weeks. With neurologic dysfunction, the patient will not be able to do all the normal activities in the same way as before the lesion, so this statement should be discussed.

Which patient is at highest risk for developing graft-versus-host disease? A. A 65-year-old man who received an autologous blood transfusion Incorrect B. A 40-year-old man who received a kidney transplant from a living donor C. A 65-year-old woman who received a pancreas and kidney from a deceased donor D. A 40-year-old woman who received a bone marrow transplant from a close relative

Answer: D A 40-year-old woman who received a bone marrow transplant from a close relative Rationale: Graft-versus-host disease occurs when an immunoincompetent patient is transfused or transplanted with immunocompetent cells. Examples include blood transfusions or the transplantation of bone marrow, fetal thymus, or fetal liver. An autologous blood transfusion is the collection and reinfusion of the individual's own blood or blood components. There is no risk for graft-versus-host disease in this situation.

The home care nurse is visiting patients in the community. Which patient is exhibiting an early warning sign of Alzheimer's disease? A. A 65-year-old male does not recognize his family members and close friends. B. A 59-year-old female misplaces her purse and jokes about having memory loss. C. A 79-year-old male is incontinent and not able to perform hygiene independently. D. A 72-year-old female is unable to locate the address where she has lived for 10 years.

Answer: D A 72-year-old female is unable to locate the address where she has lived for 10 years. Rationale: An early warning sign of Alzheimer's disease is disorientation to time and place such as geographic disorientation. Occasionally misplacing items and joking about memory loss are examples of normal forgetfulness. Impaired ability to recognize family and close friends is a clinical manifestation of middle or moderate dementia (or Alzheimer's disease). Incontinence and inability to perform self-care activities are clinical manifestations of severe or late dementia (or Alzheimer's disease).

The nurse in a primary care provider's office is assessing several patients today. Which patient is most at risk for a stroke? A. A 92-year-old female who takes warfarin (Coumadin) for atrial fibrillation. B. A 28-year-old male who uses marijuana after chemotherapy to control nausea. C. A 42-year-old female who takes oral contraceptives and has migraine headaches. D. A 72-year-old male who has hypertension and diabetes mellitus and smokes tobacco.

Answer: D A 72-year-old male who has hypertension and diabetes mellitus and smokes tobacco. Rationale: Stroke risk increases after 65 years of age. Strokes are more common in men. Hypertension is the single most important modifiable risk factor for stroke. Diabetes mellitus is a significant stroke risk factor; and smoking nearly doubles the risk of a stroke. Other risk factors include drug abuse (especially cocaine), high-dose oral contraception use, migraine headaches, and untreated heart disease such as atrial fibrillation.

A patient with intracranial pressure monitoring has a pressure of 12 mm Hg. The nurse understand that this pressure reflects A. A severe decrease in cerebral perfusion pressure B. An alteration in the production of cerebrospinal fluid C. The loss of autoregulatory control of intracranial pressure D. A normal balance between brain tissue, blood, and cerebrospinal fluid

Answer: D A normal balance between brain tissue, blood, and cerebrospinal fluid Rationale: Normal intracranial pressure (ICP) is 5 to 15 mm Hg. A sustained pressure above the upper limit is considered abnormal.

A person who injects heroine to experience the euphoria that is causes is demonstrating A. Abuse B. Addiction C. Tolerance D. Addictive Behavior

Answer: D Addictive Behavior Rationale: Addictive behavior is defined as behavior associated with maintaining an addiction.

Assessment of muscle strength of older adults cannot be compared with that of younger adults because A. Stroke is more common in older adults B. Nutritional status is better in young adults C. Most young people exercise more than older people. D. Aging leads to a decrease in muscle bulk and strength.

Answer: D Aging leads to a decrease in muscle bulk and strength. Rationale: Changes associated with aging include decreases in muscle strength and agility in relation to decreased muscle bulk.

A patient is having a transsphenoidal hypophysectomy. The nurse should provide preoperative patient teaching about what potential deficit as a result of the surgery? A. Increased heart rate B. Loss of coordination C. Impaired swallowing D. Altered sense of smell

Answer: D Altered sense of smell Rationale: Using a transsphenoidal approach to remove the pituitary gland includes a risk of damage to the olfactory cranial nerve because the cell bodies of the olfactory nerve are located in the nasal epithelium. With damage to this nerve, the sense of smell would be altered. Increased heart rate, loss of coordination, and impaired swallowing will not be potential deficits from this surgery.

Which patient may face the greatest risk of developing delirium? A. A patient with fibromyalgia whose chronic pain has recently worsened B. A patient with a fracture who has spent the night in the emergency department C. An older patient whose recent computed tomography (CT) shows brain atrophy D. An older patient who takes multiple medications to treat various health problems

Answer: D An older patient who takes multiple medications to treat various health problems Rationale: Polypharmacy is implicated in many cases of delirium, and this phenomenon is especially common among older adults. Brain atrophy, if associated with cognitive changes, is indicative of dementia. Alterations in sleep and environment, as well as pain, may cause delirium, but this is less of a risk than in an older adult who takes multiple medications.

A 74-year-old man who has right-sided extremity paralysis related to a thrombotic stroke develops constipation. Which action should the nurse take first? A. Assist the patient to the bathroom every 2 hours. B. Provide incontinence briefs to wear during the day. C. Administer a bisacodyl (Dulcolax) rectal suppository every day. D. Arrange for several servings per day of cooked fruits and vegetables.

Answer: D Arrange for several servings per day of cooked fruits and vegetables. Rationale: Patients after a stroke frequently have constipation. Dietary management includes the following: fluid intake of 2500 to 3000 mL daily, prune juice (120 mL) or stewed prunes daily, cooked fruit three times daily, cooked vegetables three times daily, and whole-grain cereal or bread three to five times daily. Patients with urinary incontinence should be assisted to the bathroom every 2 hours when appropriate. Suppositories may be ordered for short-term management if the patient does not respond to increased fluid and fiber. Incontinence briefs are indicated as a short-term intervention for urinary incontinence.

The most appropriate nursing intervention for a patient who is being treated for an acute exacerbation of chronic obstructive pulmonary disease who is not interested in quitting smoking is to A. accept the patient's decision and not intervene until the patient expresses a desire to quit. B. realize that some smokers will never quit, and trying to assist them increases the patient's frustration. C. Motivate the patient to quiet by describe how continued smoking will worsen the breathing problems. D. Ask the patient to identify the relevance, risks, and benefits of quitting and what barriers to quitting are present.

Answer: D Ask the patient to identify the relevance, risk, and benefits of quitting and what barriers to quitting are present. Rationale: The Agency for Healthcare Research and Quality (AHRQ) has issued the Clinical Practice Guideline: Treating Tobacco Use and Dependence: 2008 Update to aggressively motivate smokers and other tobacco users to quit. Five brief clinical interventions should be used at each encounter with a patient. These interventions are designed to identify tobacco users, encourage them to quit, determine their willingness to quit, assist them in quitting, and arrange for follow-up to prevent relapse.

What is the most appropriate nursing intervention to help an HIV-infected patient adhere to a treatment regimen? A. "Set up" a drug pillbox for the patient every week. B. Give the patient a video and a brochure to view and read at home. C. Tell the patient that the side effects of the drugs are bad but that they go away after a while. D. Assess the patient's routines and find adherence cues that fit into the patient's life circumstances.

Answer: D Assess the patient's routines and find adherence cues that fit into the patient's life circumstances. Rationale: The best approach to improve adherence to a treatment regimen is to learn about the patient's life and assist with problem solving within the confines of that life.

7. Association between HLA antigens and diseases is most commonly found in what disease conditions? A. Malignancies B. Infectious diseases C. Neurologic diseases D. Autoimmune disorders.

Answer: D Autoimmune disorders. Rationale: Most of the human leukocyte antigen (HLA)-associated diseases are classified as autoimmune disorders. Examples of associations between HLA types and disease include (1) that of HLA-B27 with ankylosing spondylitis, (2) those of HLA-DR2 and HLA-DR3 with systemic lupus erythematosus (SLE), and (3) those of HLA-DR3 and HLA-DR4 with diabetes mellitus.

Drugs or diseases that impair the function of the extrapyramidal system may cause loss of A. Sensations of pain and temperature. B. Regulation of the autonomic nervous system. C. Integration of somatic and special sensory system. D. Automatic movements associated with skeletal muscle activity.

Answer: D Automatic movements associated with skeletal muscle activity Rationale: A group of descending motor tracts carries impulses from the extrapyramidal system, which includes all motor systems (except the pyramidal system) concerned with voluntary movement. It includes descending pathways originating in the brainstem, basal ganglia, and cerebellum. The motor output exits the spinal cord by way of the ventral roots of the spinal nerves.

1. The function of monocytes in immunity is related to their ability to: A. Stimulate the production of T & B lymphocytes. B. Produce antibodies on exposure to foreign substances. C. Bind antigens and stimulate natural killer cell activation. D. Capture antigens by phagocytosis and present them to lymphocytes.

Answer: D Capture antigens by phagocytosis and present them to lymphocytes. Rationale: The mononuclear phagocyte system includes monocytes in the blood and macrophages found throughout the body. Mononuclear phagocytes have a critical role in the immune system. They are responsible for capturing, processing, and presenting the antigen to the lymphocytes.

Ten days after receiving a bone marrow transplant, a patient develops a skin rash on his palms and soles, jaundice, and diarrhea. What is the most likely etiology of these clinical manifestations? A. The patient is experiencing a type I allergic reaction. B. An atopic reaction is causing the patient's symptoms. C. The patient is experiencing rejection of the bone marrow. D. Cells in the transplanted bone marrow are attacking the host tissue.

Answer: D Cells in the transplanted bone marrow are attacking the host tissue Rationale: The patient's symptoms are characteristic of graft-versus-host-disease (GVHD) in which transplanted cells mount an immune response to the host's tissue. GVHD is not a type I allergic response or an atopic reaction, and it differs from transplant rejection in that the graft rejects the host rather than the host rejecting the graft.

A 52-year-old female patient was exposed to human immunodeficiency virus (HIV) 2 weeks ago through sharing needles with other substance users. What symptoms will the nurse teach the patient to report that would indicate the patient has developed an acute HIV infection? A. Cough, diarrhea, headaches, blurred vision, muscle fatigue B. Night sweats, fatigue, fever, and persistent generalized lymphadenopathy C. Oropharyngeal candidiasis or thrush, vaginal candidal infection, or oral or genital herpes D. Flu-like symptoms such as fever, sore throat, swollen lymph glands, nausea, or diarrhea

Answer: D Flu-like symptoms such as fever, sore throat, swollen lymph glands, nausea, or diarrhea Correct Rationale: Clinical manifestations of an acute infection with HIV include flu-like symptoms between 2 to 4 weeks after exposure. Early chronic HIV infection clinical manifestations are either asymptomatic or include fatigue, headache, low-grade fever, night sweats, and persistent generalized lympadenopathy. Intermediate chronic HIV infection clinical manifestations include candidal infections, shingles, oral or genital herpes, bacterial infections, Kaposi sarcoma, or oral hairy leukoplakia. Late chronic HIV infection or acquired immunodeficiency syndrome (AIDS) includes opportunistic diseases (infections and cancer).

10. What accurately describes rejeciton following transplantation? A. Hyperacute rejection can be treated with OKT3 B. Acute rejection can be treated with sirolimus or tacrolimus. C. Chronic rejection can be treated with tacrolimus or cyclosporine. D. Hyperacute reaction can usually be avoided if crossmatching is done before the transplantation.

Answer: D Hyperacute reaction can usually be avoided if crossmatching is done before the transplantation. Rationale: A positive crossmatch indicates that the recipient has cytotoxic antibodies to the donor's antigens and is an absolute contraindication to transplantation. If transplanted, the organ would undergo hyperacute rejection.

6. The nurse advises a friend who asks him to administer his allergy shots that A. It is illegal for nurses to administer injections outside of a medical setting. B. He is qualified to do it if the friend has epinephrine in an injectable syringe provided with his extract. C. Avoiding the allergens is a more effective way of controlling allergies, and allergy shots are not usually effective. D. Immunotherapy should only be administered in a setting where emergency equipment and drugs are available.

Answer: D Immunotherapy should only be administered in a setting where emergency equipment and drugs are available. Rationale: Anaphylactic reactions occur suddenly in hypersensitive patients after exposure to the offending allergen. They may occur after an allergy shot (i.e., parenteral injection). The cardinal principle in therapeutic management is speed in (1) recognition of signs and symptoms of an anaphylactic reaction, (2) maintenance of a patent airway, (3) prevention of spread of the allergen by use of a tourniquet, (4) administration of drugs, and (5) treatment for shock.

The nurse assesses a patient for signs of meningeal irritation and observes for nuchal rigidity. What indicates the presence of this sign of meningeal irritation? A. Tonic spasms of the legs B. Curling in a fetal position C. Arching of the neck and back D. Resistance to flexion of the neck

Answer: D Resistance to flexion of the neck Rationale: Nuchal rigidity is a clinical manifestation of meningitis. During assessment, the patient will resist passive flexion of the neck by the health care provider. Tonic spasms of the legs, curling in a fetal position, and arching of the neck and back are not related to meningeal irritation.

The nurse is providing postoperative care for a 58-year-old woman who has had an open cholecystectomy. The patient's daughter suggests that her mother may be alcohol dependent. The presence of which of these signs increase the likelihood that the patient may be experiencing early alcohol withdrawal symptoms? A. Flat affect, palmar erythema, and rosacea B. Drowsiness, bradycardia, and hypotension C. Seizures, hallucinations, and gross tremors D. Increased reflexes, nausea, and diaphoresis

Answer: D Increased reflexes, nausea, and diaphoresis Rationale: Early clinical manifestations of alcohol withdrawal include tremulousness, anxiety, increased heart rate and blood pressure, sweating, nausea, hyperreflexia, insomnia, and increased hyperactivity. Late manifestations include hallucinations, gross tremors, seizures, and delirium.

When planning care for a patient with a C5 spinal cord injury, which nursing diagnosis has the highest priority? A. Risk for impairment of tissue integrity caused by paralysis B. Altered patterns of urinary elimination caused by tetraplegiac C. Altered family and individual coping caused by the extent of trauma D. Ineffective airway clearance caused by high cervical spinal cord injury

Answer: D Ineffective airway clearance caused by high cervical spinal cord injury Rationale: Maintaining a patent airway is the most important goal for a patient with a high cervical fracture. Although all of these are appropriate nursing diagnoses for a patient with a spinal cord injury, respiratory needs are always the highest priority. Remember the ABCs.

A patient is admitted to the hospital with a C4 spinal cord injury after a motorcycle collision. The patient's BP is 84/50 mm Hg, his pulse is 38 beats/minute, and he remains orally intubated. The nurse determines that this pathophysiologic response is caused by A. Increase vasomotor tone after injury. B. A temporary loss of sensation and flaccid paralysis below the level of injury C. Loss of parasympathetic nervous system innervation resulting in vasoconstriction. D. Loss of sympathetic nervous system innervation resulting in peripheral vasodilation.

Answer: D Loss of sympathetic nervous system innervation resulting in peripheral vasodilation Rationale: Neurogenic shock results from loss of vasomotor tone caused by injury and is characterized by hypotension and bradycardia. Loss of sympathetic nervous system innervation causes peripheral vasodilation, venous pooling, and a decrease in cardiac output. These effects are usually associated with a cervical or high thoracic injury (T6 or higher).

A male patient suffered a diffuse axonal injury from a traumatic brain injury (TBI). He has been maintained on IV fluids for 2 days. The nurse seeks enteral feeding for this patient based on what rationale? A. Free water should be avoided. B. Sodium restrictions can be managed. C. Dehydration can be better avoided with feedings. D. Malnutrition promotes continued cerebral edema.

Answer: D Malnutirtion promotes continued cerebral edema Rationale: A patient with diffuse axonal injury is unconscious and, with increased ICP, is in a hypermetabolic, hypercatabolic state that increases the need for fuel for healing. Malnutrition promotes continued cerebral edema, and early feeding may improve outcomes when begun within 3 days after injury. Fluid and electrolytes will be monitored to maintain balance with the enteral feedings.

4. A 19-year-old man is admitted to the emergency department with a C6 spinal cord injury after a motorcycle crash. Which medication should the nurse anticipate that she will administer first? A. Enoxaparin (Lovenox) B. Metoclopramide (Reglan) C. IV immunoglobulin (Sandoglobulin) D. Methylprednisolone sodium succinate (Solu-Medrol)

Answer: D Methylprednisolone sodium succinate (Solu-Medrol) Rationale: Methylprednisolone (MP) blocks lipid peroxidation by-products and improves blood flow and reduces edema in the spinal cord. High-dose MP should be administered within 8 hours of injury. Enoxaparin is a low-molecular-weight heparin used to prevent deep vein thrombosis. Metoclopramide is used to treat delayed gastric emptying. IV immunoglobulin (Sandoglobulin) is used to treat Guillain-Barré syndrome.

In a patient with a disease that affects the myelin sheath of nerves, such as multiple sclerosis, the glial cells affected are the A. Microglia B. Astrocytes. C. Ependymal Cells D. Oligodendrocytes.

Answer: D Oligodendrocytes Rationale: Glial cell types include oligodendrocytes, astrocytes, ependymal cells, and microglia, and each has specific functions. Oligodendrocytes are specialized cells that produce the myelin sheath of nerve fibers in the central nervous system (CNS), and they are located primarily in the white matter of the CNS.

During routine assessment of a patient with Guillain-Bare syndrome, the nurse finds the patient is short of breath. The patient's respiratory distress is cause by A. Elevated protein levels in the CSF B. Immobility resulting for ascending paralysis. C. Degeneration of motor neurons in the brain-stem and spinal cord. D. Paralysis ascending to the nerves that stimulate the thoracic area.

Answer: D Paralysis ascending to the nerves that stimulate the thoracic area. Rationale: Guillain-Barré syndrome is characterized by ascending, symmetric paralysis that usually affects cranial nerves and the peripheral nervous system. The most serious complication of this syndrome is respiratory failure, which occurs as the paralysis progresses to the nerves that innervate the thoracic area.

The clinical diagnosis of dementia is based on A. CT or MRS B. Brain biopsy C. Electroencephalogram D. Patient history and cognitive assessment.

Answer: D Patient history and cognitive assessment. Rationale: The diagnosis of dementia depends on determining the cause. A thorough physical examination is performed to rule out other potential medical conditions. Cognitive testing (e.g., Mini-Mental State Examination) is focused on evaluating memory, ability to calculate, language, visual-spatial skills, and degree of alertness. Diagnosis of dementia related to vascular causes is based on the presence of cognitive loss, the presence of vascular brain lesions demonstrated by neuroimaging techniques, and the exclusion of other causes of dementia. Structural neuroimaging with computed tomography (CT) or magnetic resonance imaging (MRI) is used in the evaluation of patients with dementia. A psychologic evaluation is also indicated to determine the presence of depression.

4. In a type I hypersensitivity reaction the primary immunologic disorder appears to be A. binding of IgG to an antigen on a cell surface. B. Deposit of antigen-antibody complexes in small vessels. C. Release of cytokines used to interact with specific antigens. D. Release of chemical mediators form IgE-bound mast cells and basophils.

Answer: D Release of chemical mediators form IgE-bound mast cells and basophils. Rationale: Type I hypersensitivity reactions occur only in susceptible persons who are highly sensitized to specific allergens. Immunoglobulin E (IgE) antibodies, produced in response to the allergen, have a characteristic property of attaching to mast cells and basophils.

The nurse is providing postoperative care for a 30-year-old female patient after an appendectomy. The patient has tested positive for human immunodeficiency virus (HIV). What type of precautions should the nurse observe to prevent the transmission of this disease? A. Droplet precautions B. Contact precautions C. Airborne precautions D. Standard precautions

Answer: D Standard precautions Rationale: Standard precautions are indicated for prevention of transmission of HIV to the health care worker. HIV is not transmitted by casual contact or respiratory droplets. HIV may be transmitted through sexual intercourse with an infected partner, exposure to HIV-infected blood or blood products, and perinatal transmission during pregnancy, at delivery, or though breastfeeding.

8. The female patient has been brought to the ED with a sudden onset of a severe headache that is different from any other headache she has had previously. When considering the possibility of a stroke, which type of stroke should the nurse know is most likely occurring? A. TIA B. Embolic stroke C. Thrombotic stroke D. Subarachnoid hemorrhage

Answer: D Subarachnoid hemorrhage Headache is common in a patient who has a subarachnoid hemorrhage or an intracerebral hemorrhage. A TIA is a transient loss of neurologic function usually without a headache. A headache may occur with an ischemic embolic stroke, but severe neurologic deficits are the initial symptoms. The ischemic thrombotic stroke manifestations progress in the first 72 hours as infarction and cerebral edema increase.

Dementia is defined as a A. Syndrome that results only in memory loss. B. Disease associated with abrupt changes in behavior C. Disease that is always due to reduced blood flow to the brain. D. Syndrome characterized by a cognitive dysfunction and loss of memory

Answer: D Syndrome characterized by a cognitive dysfunction and loss of memory Rationale: Dementia is a syndrome characterized by dysfunction in or loss of memory, orientation, attention, language, judgment, and reasoning. Personality changes and behavioral problems such as agitation, delusions, and hallucinations may result.

In teaching a patient with SLE about the disorder, the nurse knows that the pathopysiology of SLE includes A. Circulating immune compolexes formed form IgG autoantibodies reacting with IgG. B. An autoimmune T-cell reaction that results in destruction of the deep dermal skin layer. C. Immunologic dysfunction leading to chronic inflammation in the cartilage and muscles. D. The production of a variety of autoantibodies directed against components of the cell nucleus.

Answer: D The production of a variety of autoantibodies directed against components of the cell nucleus. Rationale: Systemic lupus erythematosus (SLE) is characterized by the production of many autoantibodies against nucleic acids (e.g., single-and double-stranded DNA), erythrocytes, coagulation proteins, lymphocytes, platelets, and many other self-proteins. Autoimmune reactions characteristically are directed against constituents of the cell nucleus (e.g., antinuclear antibodies [ANAs]), particularly DNA. Circulating immune complexes containing antibody against DNA are deposited in the basement membranes of capillaries in the kidneys, heart, skin, brain, and joints. Complement is activated, and inflammation occurs. The overaggressive antibody response is also related to activation of B and T cells. The specific manifestations of SLE depend on which cell types or organs are involved. SLE is a type III hypersensitivity response.

Which intervention is most appropriate when communicating with a patient suffering from aphasia following a stroke? A. Present several thoughts at once so that the patient can connect the ideas. B. Ask open-ended questions to provide the patient the opportunity to speak. C. Finish the patient's sentences to minimize frustration associated with slow speech. D. Use simple, short sentences accompanied by visual cues to enhance comprehension.

Answer: D Use simple, short sentences accompanied by visual cues to enhance comprehension Rationale: When communicating with a patient with aphasia, the nurse should present one thought or idea at a time. Ask questions that can be answered with a "yes," "no," or simple word. Use visual cues and allow time for the individual to comprehend and respond to conversation.

Paralysis of lateral gaze indicates a lesion of cranial nerve A. II B. III C. IV D. VI

Answer: D VI Rationale: Cranial nerves III (oculomotor), IV (trochlear), and VI (abducens) are responsible for eye movement. The lateral rectus eye muscle is innervated by cranial nerve VI, and it is the primary muscle that is responsible for lateral eye movement.

A 31-year-old man seen at a clinic with an upper respiratory infection tells the nurse that he injects heroin IV. What screening test should the nurse encourage? A. Toxicology screening test B. Prostate-specific antigen (PSA) test C. Alcohol Use Disorders Identification Test (AUDIT) D. Human immunodeficiency virus (HIV) screening test

Answer: D Human immunodeficiency virus (HIV) screening test Rationale: Individuals who inject opioids IV are at increased risk for HIV infection and hepatitis B or C infection. In addition, drug abuse by any route increases the risk of contracting HIV because of possible risky sexual behaviors in exchange for drugs or money or because of lack of inhibition.

When caring for a patient with acute intoxication and a history of chronic alcoholism, the nurse will anticipate administering which drug? A. Morphine sulfate B. Naloxone (Narcan) C. Flumazenil (Romazicon) D. IV thiamine and glucose

Answer: D IV thiamine and glucose Rationale: Wernicke's encephalopathy, a complication of chronic alcohol abuse, is potentially preventable and reversible with the administration of IV thiamine. Patients with alcohol intoxication may also experience hypoglycemia, decreased serum magnesium, and other signs of malnutrition. For this reason, they may be treated with IV glucose, magnesium sulfate, and multivitamins. Flumazenil (Romazicon) is the reversal agent for benzodiazepines, and naloxone (Narcan) is the reversal agent for opioids. The administration of an opioid analgesic (morphine sulfate) is not indicated.

When assessing motor function of a patient admitted with a stroke, the nurse notes mild weakness of the arm demonstrated by downward drifting of the arm. How should the nurse most accurately document this finding? A. Athetosis B. Hypotonia C. Hemiparesis D. Pronator drift

Answer: D Pronator drift Rationale: Downward drifting of the arm or pronation of the palm is identified as a pronator drift. Athetosis is a slow, writhing, involuntary movement of the extremities. Hypotonia is flaccid muscle tone, and hemiparesis is weakness of one side of the body.

A 32-year-old female patient is diagnosed with diabetes insipidus after transsphenoidal resection of a pituitary adenoma. What should the nurse consider as a sign of improvement? A. Serum sodium of 120 mEq/L B. Urine specific gravity of 1.001 C. Fasting blood glucose of 80 mg/dL D. Serum osmolality of 290 mOsm/kg

Answer: D Serum osmalitiy of 290 mOsm/kg Rationale: Laboratory findings in diabetes insipidus include an elevation in serum osmolality and serum sodium and a decrease in urine specific gravity. Normal serum osmolality is 275 to 295 mOsm/kg, normal serum sodium is 135 to 145 mEq/L, and normal specific gravity is 1.003 to 1.030. Elevated blood glucose levels occur with diabetes mellitus.

The patient with an allergy to bee stings was just stung by a bee. After administering oxygen, removing the stinger, and administering epinephrine, the nurse notices the patient is hypotensive. What should be the nurse's first action? A. Administer IV diphenhydramine (Benadryl). B. Administer nitroprusside as soon as possible. C. Anticipate tracheostomy with laryngeal edema. D. Place the patient recumbent and elevate the legs.

Answer: D. Place the patient recumbent and elevate the legs. Rationale: In this emergency situation, the ABCs (airway, breathing, circulation) are being followed. For hypotension the patient should be placed in a recumbent position with the legs elevated, epinephrine will continue to be administered every 2-5 minutes, and fluids will be administered with vasopressors. Diphenhydramine is an antihistamine used to treat allergy symptoms. Anticipating a tracheostomy may occur with ongoing patient monitoring. Nitroprusside is a vasodilator and would not be used now.

The 40-year-old African American woman has had Raynaud's phenomenon for some time. She is now reporting red spots on the hands, forearms, palms, face, and lips. What other manifestations should the nurse assess for when she is assessing for scleroderma (select all that apply)? A. Calcinosis B. Weight loss C. Sclerodactyly D. Difficulty swallowing E. Weakened leg muscles

Answer:A, C, D Calcinosis, Sclerodactyly, Difficulty swallowing Rationale: This 40-year-old African American woman is at risk for scleroderma. The acronym CREST represents the clinical manifestations. C: calcinosis, painful calcium deposits in the skin; R: Raynaud's phenomenon; E: Esophageal dysfunction, difficulty swallowing; S: sclerodactyly, tightening of skin on fingers and toes; T: telangiectasia. Weight loss and weakened leg muscles are associated with polymyositis and dermatomyositis not scleroderma.

A patient is admitted to the ICU with a C7 spinal cord injury and diagnosed with a Brown-Sequard syndrome. On physical examination, the nurse would most likely find A. Upper extremity weakness only. B. Complete motor and sensory loss below C7 C. Loss of position sense and vibration in both lower extremities. D. Ipsilateral motor loss and contralateral sensory loss below C7

Answer:D Ipsilateral motor loss and contralateral sensory loss below C7 Rationale: Brown-Séquard syndrome is a result of damage to one half of the spinal cord. This syndrome is characterized by a loss of motor function, position sense, and vibratory sense, as well as by vasomotor paralysis on the same side (ipsilateral) as the injury. The opposite (contralateral) side has loss of pain and temperature sensation below the level of the injury.

Chapter 15: Infection and Human Immunodeficiency Virus Infection

Evolve & Book Questions


Conjuntos de estudio relacionados

Chapter 1 thru Chapter 6 Computer Networking Fundamentals

View Set

Principles of Finance Exam 1 Prep Questions

View Set

Exit Ticket- Day 4-9.3.12.BM.2.2 Demonstrate ethical behaviors in the workplace by Explaining the nature of business ethics and Identifying and explaining where an action is legal but unethical.

View Set

Macroeconomics chapter 13 Fiscal Policy practice part 2

View Set